All Questions on eye/ear/pain/PVD i could find

Réussis tes devoirs et examens dès maintenant avec Quizwiz!

shine light on eye artery it will ________-

light up

consensual response

reaction of both pupils when one eye is exposed to greater intensity of light than the other

main risk factor for cataracts

smoking

3. Notify the health care provider (HCP).

750. The nurse is caring for a client following enucleation and notes the presence of brightred drainage on the dressing. Which action should the nurse take at this time?

The nurse is admitting a 68-year-old preoperative patient with a suspected abdominal aortic aneurysm (AAA). The medication history reveals that the patient has been taking warfarin (Coumadin) on a daily basis. Based on this history and the patient's admission diagnosis, the nurse should prepare to administer which medication? a. Vitamin K b. Cobalamin c. Heparin sodium d. Protamine sulfate

*Correct Answer: A* Coumadin is a Vitamin K antagonist anticoagulant that could cause excessive bleeding during surgery if clotting times are not corrected before surgery. For this reason, vitamin K is given as the antidote for warfarin (Coumadin).

The client asks the nurse how glaucoma develops. What is the best response by the nurse? Select all that apply. 1. "Having diabetes is a risk factor in the development of glaucoma." 2. "Glaucoma exists when the pressure in your eye is high enough to cause optic nerve damage." 3. "High blood pressure can certainly lead to the development of glaucoma." 4. "Having blue eyes is a risk factor in the development of glaucoma." 5. "Drugs like antihistamines and antidepressants can contribute to glaucoma."

1. "Having diabetes is a risk factor in the development of glaucoma." 2. "Glaucoma exists when the pressure in your eye is high enough to cause optic nerve damage." 3. "High blood pressure can certainly lead to the development of glaucoma." 5. "Drugs like antihistamines and antidepressants can contribute to glaucoma."

A client is prescribed a beta-adrenergic blocker for treatment of glaucoma. The nurse provides specific information on administration technique to avoid which adverse effects? Select all that apply. 1. Bronchoconstriction 2. Cardiac dysrhythmias 3. Low blood pressure 4. Rash 5. Vomiting

1. Bronchoconstriction 2. Cardiac dysrhythmias 3. Low blood pressure

Which of the following best describes open-angle glaucoma? 1. It is bilateral with a slow onset. 2. It is less common than closed-angle glaucoma. 3. It is accompanied by eye pain. 4. It is unilateral with a rapid onset.

1. It is bilateral with a slow onset.

The client is preparing for an eye examination. Which of the following classification of drugs will be used? 1. Mydriatics 2. Vasoconstrictors 3. Prostaglandins 4. Beta-adrenergic blockers

1. Mydriatics

The client wears contact lenses and has been prescribed eyedrops for glaucoma. What will the best education by the nurse include with regard to contact lenses? 1. Remove lenses before instilling eyedrops; do not reinsert lenses for 15 minutes. 2. Instill the drops with the contacts in as long as they are the hard kind of contacts. 3. Eyeglasses must be worn for as long as the client must have the eyedrops. 4. Instill the drops with the contacts in as long as they are the soft kind of contacts.

1. Remove lenses before instilling eyedrops; do not reinsert lenses for 15 minutes

A client has been prescribed brinzolamide (Azopt). Which findings would the nurse discuss with the prescriber before administering this drug? Select all that apply. 1. The client's last potassium level was low. 2. The client is a vegetarian. 3. The client is allergic to sulfa. 4. The client has a history of atopic dermatitis. 5. The client's father had a myocardial infarction at age 35.

1. The client's last potassium level was low. 3. The client is allergic to sulfa

The nurse is reviewing the laboratory test results for a 68-year-old patient whose warfarin (Coumadin) therapy was terminated during the preoperative period. The nurse concludes that the patient is in the most stable condition for surgery after noting which INR (international normalized ratio) result? 1.0 1.8 2.7 3.4

1.0 The therapeutic range for INR is 2.0 to 3.0 for many clinical diagnoses. The larger the INR number, the greater the amount of anticoagulation. For this reason, the safest value before surgery is 1.0, meaning that the anticoagulation has been reversed.

normal near vision acuity

14/14

A client presents to the emergency department with symptoms of closed-angle glaucoma. Intraocular pressure is measured at 30 mmHg. Which assessment findings would the nurse anticipate? Select all that apply. 1. Absence of eye pain 2. Headache 3. Bloodshot eyes 4. Vomiting 5. Bruising around the orbit

2. Headache 3. Bloodshot eyes 4. Vomiting

The drug most likely to be used to soften or eliminate cerumen is 1. a steroid. 2. a wax softener. 3. an antifungal agent. 4. a local analgesic.

2. a wax softener.

normal distant acuity

20/20

The client is to receive eyedrops for glaucoma. What is the correct method of administration? 1. Place the drop in the center of the eye. 2. Place the drop so it falls on the white part of the eye. 3. Place the drop in the conjunctival sac below the eye. 4. Turn the head to the side so that the drop flows to the outer corner.

3. Place the drop in the conjunctival sac below the eye

The primary treatment for mastoiditis is 1. topical antibiotics. 2. topical steroids. 3. systemic antibiotics. 4. Systemic steroids.

3. systemic antibiotics.

10. The nurse knows that which of the following are distinct landmarks of the tympanic membrane. Select all that apply. Incus_1___ Stapes__2__ Umbo__3__ Cone of light_4__ Handle of malleus___5_ Pars flaccida_6___ Pars tensa__7__

34567

Which of the following is an adverse effect of prostaglandins, such as latanoprost (Xaltan)? 1. Hypotension 2. Nausea 3. Tachycardia 4. Eye pain

4. Eye pain

A client has had several measurements of intraocular pressure. Which measurements would the nurse interpret as indicating need for pharmacologic intervention? Select all that apply. 1. A single reading of 14 mmHg 2. A consistent reading trending between 15 and 16 mmHg. 3. A single reading of 19 mmHg. 4. Readings consistently higher than 21 mmHg. 5. A single reading of 32 mmHg.

4. Readings consistently higher than 21 mmHg. 5. A single reading of 32 mmHg.

The client has glaucoma. Which assessment finding indicates to the nurse that a client's medical regimen could have contributed to onset of glaucoma? 1. Taking a beta blocker to treat hypertension 2. Occasional use of antihistamines for allergies 3. Taking glucocorticoids to treat arthritis 4. Regular use of an antidepressant drug

4. Regular use of an antidepressant drug

The client has excess cerumen in his ears. What will the best plan by the nurse include as to the safe removal of the cerumen? 1. Use a sterile Q-tip to remove cerumen. 2. Instill 2% acetic acid in each ear. 3. Take hot showers to facilitate drainage. 4. Use warm water and a bulb syringe.

4. Use warm water and a bulb syringe.

The nurse is counseling a client with glaucoma. The nurse explains that, if left untreated, the condition can lead to 1. myopia. 2. nearsightedness. 3. diabetes mellitus. 4. blindness.

4. blindness.

what age group uses Jaeger Card-hand

40 and over

Cataract surgery results in aphakia. Which of the following statements best describes this term? A Absence of the crystalline lens B A "keyhole" pupil C Loss of accommodation D Retinal detachment

A. Aphakia means without lens, a keyhole pupil results from iridectomy. Loss of accommodation is a normal response to aging. A retinal detachment is usually associated with retinal holes created by vitreous traction.

The nurse educates a primary HTN patient on lifestyle changes. Which ones should be included in her teaching? A. consume more fruits/veggies B. Monitor/lose weight C. Limit alchoholic drinks to 3 per day or less D. Regular exercise (walking) E. Limit sodium intake to 3200 mg per day

A, B, D

4. The client reports ear pain and a popping sensation in the left ear. What question should the nurse ask next? A. Are you experiencing any ringing or crackling in your ears? B. Do you have a sore throat or teeth and gum problems? C. Are you having trouble hearing high pitched sounds? D. Do you feel like you or the room is spinning?

b

7. A client complains of the feeling that "something is in my eye." What structures should the nurse inspect to rule out a foreign object? A. Iris and pupil? B. Palpebral conjunctive C. Lacrimal apparatus D. Cornea and the lens

b

1. A nurse is caring for a patient with a diagnosis of deep venous thrombosis (DVT). The patient has an order to receive 30 mg enoxaparin (Lovenox). Which injection site should the nurse use to administer this medication safely? Buttock, upper outer quadrant Abdomen, anterior-lateral aspect Back of the arm, 2 inches away from a mole Anterolateral thigh, with no scar tissue nearby

Abdomen, anterior-lateral aspect

What is the best way to schedule medication for a client with constant pain?

Around-the-clock

The nurse is caring for a client with lower extremity peripheral arterial disease. Which statement made by the client regarding self- management requires further health teaching? A. "I need to quit smoking as soon as I can." B. "I will elevate my legs above the level of my heart." C. "I will use a heating pad to promote circulation." D. "I will avoid crossing my legs at all times."

B

The majority of people over 65 have PVD? A. True B. False

B. False

Which teaching point does the nurse include for a client with peripheral arterial disease (PAD)? A. "Elevate your legs above heart level to prevent swelling." B. "Inspect your legs daily for brownish discoloration around the ankles." C. "Walk to the point of leg pain, then rest, resuming when pain stops." D. "Apply a heating pad to the legs if they feel cold."

C Exercise may improve arterial blood flow by building collateral circulation; instruct the client to walk until the point of claudication, stop and rest, and then walk a little farther.

A client is being tapered off opioids and the nurse is watchful for signs of withdrawal. What is one of the first signs of withdrawal?

Diaphoresis

A client appears upset and tearful, but denies pain and refuses pain medication, because "my sibling is a drug addict and has ruined out lives." What is the priority intervention for this client?

Encourage expression of fears on past experiences

In caring for a young child with pain, which assessment tool is the most useful?

Faces pain-rating scale

As the charge nurse, you are reviewing the charts of clients who were assigned to a newly graduated RN. The RN has correctly charted dose and time of medication, but there is no documentation regarding non-pharmaceutical measures. What action should you take first?

Give praise for the correct dose and time and discuss the deficits in charting.

Buerger's disease is characterized by all of the following except: a. Arterial thrombosis formation and occlusion b. Lipid deposits in the arteries c. Redness or cyanosis in the limb when it is dependent d. Venous inflammation and occlusion

Lipid deposits in the arteries

With peripheral arterial insufficiency, leg pain during rest can be reduced by: a. Elevating the limb above heart level b. Lowering the limb so it is dependent c. Massaging the limb after application of cold compresses d. Placing the limb in a plane horizontal to the body

Lowering the limb so it is dependent

For a client who is taking aspirin, which laboratory value should be reported to the physician?

PT 14 seconds

The nurse would determine that a postoperative patient is not receiving the beneficial effects of enoxaparin (Lovenox) after noting what during a routine shift assessment? Generalized weakness and fatigue Crackles bilaterally in the lung bases Pain and swelling in lower extremity Abdominal pain with decreased bowel sound

Pain and swelling in lower extremity Correct Enoxaparin is a low-molecular-weight heparin used to prevent the development of deep vein thromboses (DVTs) in the postoperative period. Pain and swelling in the lower extremity can indicate development of DVT and therefore may signal ineffective medication therapy.

Which route of administration is preferred if immediate analgesia and rapid titration are necessary?

Patient-controlled analgesia (PCA)

17.ID: 809565397 A patient was just diagnosed with acute arterial ischemia in the left leg secondary to atrial fibrillation. Which early clinical manifestation must be reported to the physician immediately to save the patient's limb? Paralysis Paresthesia Crampiness Referred pain

Paresthesia The physician must be notified immediately if any of the six Ps of acute arterial ischemia occur to prevent ischemia from quickly progressing to tissue necrosis and gangrene. The six Ps are paresthesia, pain, pallor, pulselessness, and poikilothermia, with paralysis being a very late sign indicating the death of nerves to the extremity. Crampy leg sensation is more common with varicose veins. The pain is not referred.

Intravenous heparin therapy is ordered for a client. While implementing this order, a nurse ensures that which of the following medications is available on the nursing unit? A Vitamin K B Aminocaproic acid C Potassium chloride D Protamine sulfate

Protamine sulfate

Which of the following nursing diagnoses is the priority diagnosis for a patient with Ménière's disease? Disturbed Sensory Perception: Auditory Disturbed Sleep Pattern Risk for Trauma Social Isolation

Risk for Trauma Rationale: The risk for trauma in patients with inner ear disorders, such as Ménière's disease, is great. Disturbed Sensory Perception: Auditory is a priority nursing diagnosis for patients with hearing loss. Sleep Pattern Disturbance is a priority nursing diagnosis for patients with tinnitus. Risk for Trauma is the priority nursing diagnosis for patients with inner ear disturbances causing balance disorders. Social Isolation is a priority diagnosis for patients with otosclerosis or presbycusis.

Family members are encouraging your client to "tough it out" rather than run the risk of becoming addicted to narcotics. The client is stoically abiding by the family's wishes. Priority nursing interventions for this client should target which dimension of pain?

Sociocultural

What is a priority nursing intervention in the care of a patient with a diagnosis of chronic venous insufficiency (CVI)? Application of topical antibiotics to venous ulcers Maintaining the patient's legs in a dependent position Administration of oral and/or subcutaneous anticoagulants Teaching the patient the correct use of compression stockings

Teaching the patient the correct use of compression stockings CVI requires conscientious and consistent application of compression stockings. Anticoagulants are not necessarily indicated and antibiotics, if required, are typically oral or IV, not topical. The patient should avoid prolonged positioning with the limb in a dependent position.

In applying the principles of pain treatment, what is the first consideration?

The client must be believed about perceptions of own pain.

A patient was just diagnosed with acute arterial ischemia in the left leg secondary to atrial fibrillation. Which early clinical manifestation must be reported to the physician immediately to save the patient's limb? Paralysis Paresthesia Crampiness Referred pain

The physician must be notified immediately if any of the six Ps of acute arterial ischemia occur to prevent ischemia from quickly progressing to tissue necrosis and gangrene. The six Ps are paresthesia, pain, pallor, pulselessness, and poikilothermia, with paralysis being a very late sign indicating the death of nerves to the extremity. Crampy leg sensation is more common with varicose veins. The pain is not referred.

In educating clients about non-pharmaceutical alternatives, which topic could you delegate to an experienced LPN/LVN, who will function under your continued support and supervision?

Use of heat and cold applications

Which technique is considered the gold standard for diagnosing DVT? a. Ultrasound imaging b. Venography c. MRI d. Doppler flow study

Venography

1. Which of the following muscle groups in the eye is responsible for straight movement? A. Superior rectus, inferior rectus, medial rectus, later rectus? B. Superior rectus, inferior rectus, superior oblique, inferior oblique C. Superior oblique, inferior oblique, medial rectus, inferior rectus D. Superior oblique, inferior oblique, medical rectus, superior rectus

a

5. Which of the following statements regarding hearing loss is correct? A. Many medication can produce loss of hearing B. Native Americans have a lower rate of ear disease C. Late onset of hearing loss affect women more than men D. Early onset of hearing loss has a genetic component

a

6. The nurse recognizes that generalized redness of the eye is indicative of what condition? A. Conjunctivitis B. Pinguecula C. Episcleritis D. Blepharitis

a

A nurse is assessing the neurovascular status of a client who returned to the surgical nursing unit 4 hours ago after undergoing aortoiliac bypass graft. The affected leg is warm, and the nurse notes redness and edema. The pedal pulse is palpable and unchanged from admission. The nurse interprets that the neurovascular status is: a. Normal because of the increased blood flow through the leg b. Slightly deteriorating and should be monitored for another hour c. Moderately impaired, and the surgeon should be called. d. Adequate from the arterial approach, but venous complications are arising.

a. An expected outcome of surgery is warmth, redness, and edema in the surgical extremity because of increased blood flow. Options 2, 3, and 4 are incorrect interpretations.

A nurse is caring for a client who had a percutaneous insertion of an inferior vena cava filter and was on heparin therapy before surgery. The nurse would inspect the surgical site most closely for signs of: a. Thrombosis and infection b. Bleeding and infection c. Bleeding and wound dehiscence. d. Wound dehiscence and evisceration.

b. After inferior vena cava insertion, the nurse inspects the surgical site for bleeding and signs and symptoms of infection. Otherwise, care is the same as for any post-op client

A client comes to the outpatient clinic and tells the nurse that he has had legs pains that begin when he walks but cease when he stops walking. Which of the following conditions would the nurse assess for? a. An acute obstruction in the vessels of the legs b. Peripheral vascular problems in both legs c. Diabetes d. Calcium deficiency

b. Intermittent claudication is a condition that indicates vascular deficiencies in the peripheral vascular system. If an obstruction were present, the leg pain would persist when the client stops walking. Low calcium levels may cause leg cramps but would not necessarily be related to walking.

A client comes to the outpatient clinic and tells the nurse that he has had legs pains that begin when he walks but cease when he stops walking. Which of the following conditions would the nurse assess for? a. An acute obstruction in the vessels of the legs b. Peripheral vascular problems in both legs c. Diabetes d. Calcium deficiency

b. Intermittent claudication is a condition that indicates vascular deficiencies in the peripheral vascular system. If an obstruction were present, the leg pain would persist when the client stops walking. Low calcium levels may cause leg cramps but would not necessarily be related to walking.

A nurse is assessing a client with an abdominal aortic aneurysm. Which of the following assessment findings by the nurse is probably unrelated to the aneurysm? a. Pulsatile abdominal mass b. Hyperactive bowel sounds in that area c. Systolic bruit over the area of the mass d. Subjective sensation of "heart beating" in the abdomen.

b. Not all clients with abdominal aortic aneurysms exhibit symptoms. Those who do describe a feeling of the "heart beating" in the abdomen when supine or be able to feel the mass throbbing. A pulsatile mass may be palpated in the middle and upper abdomen. A systolic bruit may be auscultated over the mass. Hyperactive bowel sounds are not related specifically to an abdominal aortic aneurysm.

Which technique is considered the gold standard for diagnosing DVT? a. Ultrasound imaging b. Venography c. MRI d. Doppler flow study

b. Venography

A client who has been receiving heparin therapy also is started on warfarin sodium (coumadin). The client asks the nurse why both medications are being administered. In formulating a response, the nurse incorporates the understanding that warfarin sodium: a. Stimulates the breakdown of specific clotting factors by the liver, and it takes 2-3 days for this is exhibit an anticoagulant effect. b. Inhibits synthesis of specific clotting factors in the liver, and it takes 3 to 4 days for this medication to exert an anticoagulation effect. c. Stimulates production of the body's own thrombolytic substances, but it takes 2-4 days for it to begin. d. Has the same mechanism action of heparin, and the crossover time is needed for the serum level of warfarin sodium to be therapeutic.

b. Warfarin sodium works in the liver and inhibits synthesis of four vitamin K-dependent clotting factors (X, IX, VII, and II), but it takes 3 to 4 days before the therapeutic effect of warfarin is exhibited.

9. Which of these demonstrates proper procedure when performing the Rinne test? A. Strike the tuning fork; hold it to the ear, than mastoid process? B. Strike the tuning fork: hold it to one ear, than the other? C. Strike the tuning fork: hold it to the mastoid process, then the ear? D. Strike the tuning fork: hold it to the ear, then the top of the head?

c

Which of the following characteristics is typical of the pain associated with DVT? a. Dull ache b. No pain c. Sudden onset d. Tingling

c. DVT is associated with deep leg pain of sudden onset, which occurs secondary to the occlusion. A dull ache is more commonly associated with varicose veins. A tingling sensation is associated with an alteration in arterial blood flow. If the thrombus is large enough, it will cause pain.

Morphine 10 mg IV every 4 to 6 hours prn is ordered for a patient with a pancreatic tumor who has a distant history of opioid abuse. After 3 days of receiving the morphine every 6 hours, the patient tells the nurse that the medication is needed more frequently to control the pain. The best initial action by the nurse is to a. administer the morphine every 4 hours as needed. b. consult with the doctor about initiating an appropriate weaning protocol for the morphine. c. remind the patient that the previous substance abuse increases the risk for addiction. d. use alternative therapies such as heat or cold.

correct Answer: A Rationale: These patient data indicate that tolerance for the morphine is developing and more frequent administration is needed to maintain pain control. A weaning protocol is not indicated, since the patient still has the pancreatic tumor and there is no indication that the physiologic basis of the pain has changed. Although the patient may be at risk for addiction, adequate pain management is the priority at present. Alternative therapies may be a useful adjuvant to the morphine but should not be the first nursing action. Cognitive Level: Application Text Reference: p. 148 Nursing Process: Implementation NCLEX: Physiological Integrity

2. What signs or symptoms should the nurse recognize as being associated with inner ear dysfunction? A. Pain of discomfort? B. Ear drainage? C. Blue tinged ear drum? D. Dizziness?

d

A client is admitted with a venous stasis leg ulcer. A nurse assesses the ulcer, expecting to note that the ulcer: a. Has a pale colored base b. Is deep, with even edges c. Has little granulation tissue d. Has brown pigmentation around it.

d. Venous leg ulcers, also called stasis ulcers, tend to be more superficial than arterial ulcers, and the ulcer bed is pink. The edges of the ulcer are uneven, and granulation tissue is evident. The skin has a brown pigmentation from accumulation of metabolic waste products resulting from venous stasis. The client also exhibits peripheral edema. (options 1, 2, and 3 is due to tissue malnutrition; and thus us an arterial problem)

The nurse in the ophthalmologists office is instructing the teenager about wearing and caring for contact lenses. Which of the following statements, if made by the client to the nurse, would indicate that the teaching has been successful? "I'll be able to sleep in these." "Use boiled water to clean the lenses." "The lenses will cause some pain during the first few days I wear them." "Even when my friends ask to try them on, I'll tell them 'No.'"

"Even when my friends ask to try them on, I'll tell them 'No.'" Rationale: Contact lens care includes removing lenses before sleep; using cleaning and wetting solutions recommended by the ophthalmologist or the manufacturer; removing the lenses and contacting the ophthalmologist if pain occurs; and, not sharing contact lenses or allowing another person to "try on" the lenses.

Which of the following statements by a patient with newly prescribed contact lenses indicates a need for additional teaching? "I think it is so cool that my best friend, Taylor, and I have the same prescription. She has blue lenses and I have brown lenses, so we'll be able to trade to see what we would look like with different eyes." "If I run out of cleaning and wetting solution, I'll have to wait until I get a new supply before I'll be able to wear my lenses again." "I'll follow the manufacturer's guide to tell me when I need to take them out." "If I have eye redness or pain in my eyes, I'll take out my lenses and call the ophthalmologist first thing."

"I think it is so cool that my best friend, Taylor, and I have the same prescription. She has blue lenses and I have brown lenses, so we'll be able to trade to see what we would look like with different eyes." Rationale: Manufacturer's recommendations indicate to the wearer proper length of time for wear, cleaning, and storing. Patients are instructed to use cleaning and wetting solutions recommended by the optometrist, ophthalmologist, or lens manufacturer. Patients are instructed not to use water or homemade solutions for wetting or cleaning lenses. If eye redness, tearing, vision loss, or pain occurs, the patient should remove the lenses and contact the eye care professional as soon as possible. Lenses should not be shared or "tried on" by another person.

The 77-year-old male smoker has developed cataracts on both eyes, and is awaiting surgery. The cataracts have prevented the patient from driving, reading, and seeing television. Which of the following statements would be appropriate for the nurse in the preoperative holding area to communicate to the patient? "Did your eye doctor explain to you that you could choose to use eye drops six times a day to dissolve the cataract?" "You are wise to have both eyes done at the same time." "If you were a non-smoker, you would never have developed cataracts." "Most patients undergoing cataract surgery do well and enjoy the improved vision."

"Most patients undergoing cataract surgery do well and enjoy the improved vision." Rationale: Surgical removal is the only treatment used at this time for cataracts; no medical treatment is available to prevent or treat them. Surgical removal of the cataract and lens is indicated when the cataract has developed to the point that vision and activities of daily living are affected. If the patient presents with bilateral cataracts, surgery is only performed on one eye at a time. Complications from cataract surgery are unusual, and occur in less than 1% of the surgeries.

A family member asks you, "Why can't you give more medicine? He is still having a lot of pain." What is your best response?

"Please tell him that I will be right there to check of him."

A 40-year-old man tells the nurse he has a diagnosis for the color and temperature changes of his limbs but can't remember the name of it. He says he must stop smoking and avoid trauma and exposure of his limbs to cold temperatures to get better. This description should allow the nurse to ask the patient if he has which diagnosis? a Buerger's disease b Venous thrombosis c Acute arterial ischemia d Raynaud's phenomenon

*Correct Answer: A* Rationale: Buerger's disease is a nonatherosclerotic, segmental, recurrent inflammatory disorder of small and medium-sized veins and arteries of upper and lower extremities leading to color and temperature changes of the limbs, intermittent claudication, rest pain, and ischemic ulcerations. It primarily occurs in men younger than 45 years old with a long history of tobacco and/or marijuana use. Buerger's disease treatment includes smoking cessation, trauma and cold temperature avoidance, and a walking program. Venous thrombosis is the formation of a thrombus in association with inflammation of the vein. Acute arterial ischemia is a sudden interruption in arterial blood flow to a tissue caused by embolism, thrombosis, or trauma. Raynaud's phenomenon is characterized by vasospasm-induced color changes of the fingers, toes, ears, and nose.

The patient has CVI and a venous ulcer. The unlicensed assistive personnel (UAP) decides to apply compression stockings because that is what these patients always have ordered. What assessment by the nurse would cause the application of compression stockings to harm the patient? a Rest pain b High blood pressure c Elevated blood sugar d Dry, itchy, flaky skin

*Correct Answer: A* Rationale: Rest pain occurs as peripheral artery disease (PAD) progresses and involves multiple arterial segments. Compression stockings should not be used on patients with PAD. Elevated blood glucose, possibly indicating uncontrolled diabetes mellitus, and hypertension may or may not indicate arterial problems. Dry, itchy, flaky skin indicates venous insufficiency. The RN should be the one to obtain the order and instruct the UAP to apply compression stockings if they are ordered.

When the patient is being examined for venous thromboembolism (VTE) in the calf, what diagnostic test should the nurse expect to teach the patient about first? a Duplex ultrasound b Contrast venography c Magnetic resonance venography d Computed tomography venography

*Correct Answer: A* Rationale: The duplex ultrasound is the most widely used test to diagnose VTE. Contrast venography is rarely used now. Magnetic resonance venography is less accurate for calf veins than pelvic and proximal veins. Computed tomography venography may be used but is invasive and much more expensive than the duplex ultrasound.

Assessment of a patient's peripheral IV site reveals that phlebitis has developed over the past several hours. Which intervention should the nurse implement first? a. Remove the patient's IV catheter. Correct b. Apply an ice pack to the affected area. c. Decrease the IV rate to 20 to 30 mL/hr. d. Administer prophylactic anticoagulants.

*Correct Answer: A* Rationale: The priority intervention for superficial phlebitis is removal of the offending IV catheter. Decreasing the IV rate is insufficient. Anticoagulants are not normally required, and warm, moist heat is often therapeutic.

Which person should the nurse identify as having the highest risk for abdominal aortic aneurysm? a A 70-year-old male, with high cholesterol and hypertension Correct b A 40-year-old female with obesity and metabolic syndrome c A 60-year-old male with renal insufficiency who is physically inactive d A 65-year-old female with hyperhomocysteinemia and substance abuse

*Correct Answer: A* The most common etiology of descending abdominal aortic aneurysm (AAA) is atherosclerosis. Male gender, age 65 years or older, and tobacco use are the major risk factors for AAAs of atherosclerotic origin. Other risk factors include the presence of coronary or peripheral artery disease, high blood pressure, and high cholesterol.

A 73-year-old man with dementia has a venous ulcer related to chronic venous insufficiency. The nurse should provide education on which type of diet for this patient and his caregiver? a Low-fat diet b High-protein diet c Calorie-restricted diet d High-carbohydrate diet

*Correct Answer: B* A patient with a venous ulcer should have a balanced diet with adequate protein, calories, and micronutrients; this type of diet is essential for healing. Nutrients most important for healing include protein, vitamins A and C, and zinc. Foods high in protein (e.g., meat, beans, cheese, tofu), vitamin A (green leafy vegetables), vitamin C (citrus fruits, tomatoes, cantaloupe), and zinc (meat, seafood) must be provided. Restricting fat or calories is not helpful for wound healing or in patients of normal weight. For overweight individuals with no active venous ulcer, a weight-loss diet should be considered.

A 67-year-old man with peripheral artery disease is seen in the primary care clinic. Which symptom reported by the patient would indicate to the nurse that the patient is experiencing intermittent claudication? a Patient complains of chest pain with strenuous activity. b Patient says muscle leg pain occurs with continued exercise. c Patient has numbness and tingling of all his toes and both feet. d Patient states the feet become red if he puts them in a dependent position.

*Correct Answer: B* Intermittent claudication is an ischemic muscle ache or pain that is precipitated by a consistent level of exercise, resolves within 10 minutes or less with rest, and is reproducible. Angina is the term used to describe chest pain with exertion. Paresthesia is the term used to describe numbness or tingling in the toes or feet. Reactive hyperemia is the term used to describe redness of the foot; if the limb is in a dependent position the term is dependent rubor.

A patient was just diagnosed with acute arterial ischemia in the left leg secondary to atrial fibrillation. Which early clinical manifestation must be reported to the physician immediately to save the patient's limb? a Paralysis b Paresthesia c Crampiness d Referred pain

*Correct Answer: B* Rationale: The physician must be notified immediately if any of the six Ps of acute arterial ischemia occur to prevent ischemia from quickly progressing to tissue necrosis and gangrene. The six Ps are paresthesia, pain, pallor, pulselessness, and poikilothermia, with paralysis being a very late sign indicating the death of nerves to the extremity. Crampy leg sensation is more common with varicose veins. The pain is not referred.

A 62-year-old Hispanic male patient with diabetes mellitus has been diagnosed with peripheral artery disease (PAD). The patient is a smoker and has a history of gout. What should the nurse focus her teaching on to prevent complications for this patient? a Gender b Smoking c Ethnicity d Co-morbidities

*Correct Answer: B* Rationale: Smoking is the most significant factor for this patient. PAD is a marker of advanced systemic atherosclerosis. Therefore tobacco cessation is essential to reduce PAD progression, CVD events, and mortality. Diabetes mellitus and hyperuricemia are also risk factors. Being male or Hispanic are not risk factors for PAD.

The nurse is caring for a newly admitted patient with vascular insufficiency. The patient has a new order for enoxaparin (Lovenox) 30 mg subcutaneously. What should the nurse do to correctly administer this medication? a. Spread the skin before inserting the needle. b. Leave the air bubble in the prefilled syringe. c. Use the back of the arm as the preferred site. d. Sit the patient at a 30-degree angle before administration.

*Correct Answer: B* Rationale: The nurse should not expel the air bubble from the prefilled syringe because it should be injected to clear the needle of medication and avoid leaving medication in the needle track in the tissue.

The first priority of collaborative care of a patient with a suspected acute aortic dissection is to a. reduce anxiety. b. control blood pressure. c. monitor for chest pain. d. increase myocardial contractility.

*Correct answer: b* Rationale: The initial goals of therapy for acute aortic dissection without complications are blood pressure (BP) control and pain management. BP control reduces stress on the aortic wall by reducing systolic BP and myocardial contractility.

A patient with varicose veins has been prescribed compression stockings. How should the nurse teach the patient to use these? a. "Try to keep your stockings on 24 hours a day, as much as possible." b. "While you're still lying in bed in the morning, put on your stockings." c. "Dangle your feet at your bedside for 5 minutes before putting on your stockings." d. "Your stockings will be most effective if you can remove them for a few minutes several times a day."

*Correct Answer: B* Rationale: The patient with varicose veins should apply stockings in bed, before rising in the morning. Stockings should not be worn continuously, but they should not be removed several times daily. Dangling at the bedside prior to application is likely to decrease their effectiveness.

The patient had aortic aneurysm repair. What priority nursing action will the nurse use to maintain graft patency? a Assess output for renal dysfunction. b Use IV fluids to maintain adequate BP. c Use oral antihypertensives to maintain cardiac output. d Maintain a low BP to prevent pressure on surgical site

*Correct Answer: B* Rationale: The priority is to maintain an adequate BP (determined by the surgeon) to maintain graft patency. A prolonged low BP may result in graft thrombosis, and hypertension may cause undue stress on arterial anastomoses resulting in leakage of blood or rupture at the suture lines, which is when IV antihypertensives may be used. Renal output will be assessed when the aneurysm repair is above the renal arteries to assess graft patency, not maintain it.

A female patient with critical limb ischemia has had peripheral artery bypass surgery to improve her circulation. What care should the nurse provide on postoperative day 1? a Keep the patient on bed rest. b Assist the patient with walking several times. c Have the patient sit in the chair several times. d Place the patient on her side with knees flexed.

*Correct Answer: B* Rationale: To avoid blockage of the graft or stent, the patient should walk several times on postoperative day 1 and subsequent days. Having the patient's knees flexed for sitting in a chair or in bed increase the risk of venous thrombosis and may place stress on the suture lines.

A postoperative patient asks the nurse why the physician ordered daily administration of enoxaparin (Lovenox). Which reply by the nurse is most appropriate? a. "This medication will help prevent breathing problems after surgery, such as pneumonia." b. "This medication will help lower your blood pressure to a safer level, which is very important after surgery." c. "This medication will help prevent blood clots from forming in your legs until your level of activity, such as walking, returns to normal." d. "This medication is a narcotic pain medication that will help take away any muscle aches caused by positioning on the operating room table."

*Correct Answer: C* Enoxaparin is an anticoagulant that is used to prevent DVTs postoperatively. All other explanations/options do not describe the action/purpose of enoxaparin.

The nurse would determine that a postoperative patient is not receiving the beneficial effects of enoxaparin (Lovenox) after noting what during a routine shift assessment? a. Generalized weakness and fatigue b. Crackles bilaterally in the lung bases c. Pain and swelling in lower extremity d. Abdominal pain with decreased bowel sounds

*Correct Answer: C* Rationale: Enoxaparin is a low-molecular-weight heparin used to prevent the development of deep vein thromboses (DVTs) in the postoperative period. Pain and swelling in the lower extremity can indicate development of DVT and therefore may signal ineffective medication therapy.

The nurse is caring for a preoperative patient who has an order for vitamin K by subcutaneous injection. The nurse should verify that which laboratory study is abnormal before administering the dose? a. Hematocrit (Hct) b. Hemoglobin (Hgb) c. Prothrombin time (PT) d. Partial thromboplastin time (PTT)

*Correct Answer: C* Rationale: Vitamin K counteracts hypoprothrombinemia and/or reverses the effects of warfarin (Coumadin) and thus decreases the risk of bleeding. High values for either the prothrombin time (PT) or the international normalized ratio (INR) demonstrates the need for this medication.

The nurse is caring for a patient who has been receiving warfarin (Coumadin) and digoxin (Lanoxin) as treatment for atrial fibrillation. Because the warfarin has been discontinued before surgery, the nurse should diligently assess the patient for which complication early in the postoperative period until the medication is resumed? a. Decreased cardiac output b. Increased blood pressure c. Cerebral or pulmonary emboli d. Excessive bleeding from incision or IV sites

*Correct Answer: C* Rationale: Warfarin is an anticoagulant that is used to prevent thrombi from forming on the walls of the atria during atrial fibrillation. Once the medication is terminated, thrombi could again form. If one or more thrombi detach from the atrial wall, they could travel as cerebral emboli from the left atrium or pulmonary emboli from the right atrium.

A 32-year-old female is prescribed diltiazem (Cardizem) for Raynaud's phenomenon. To evaluate the patient's expected response to this medication, what is most important for the nurse to assess? a Improved skin turgor b Decreased cardiac rate c Improved finger perfusion d Decreased mean arterial pressure

*Correct Answer: C* Raynaud's phenomenon is an episodic vasospastic disorder of small cutaneous arteries, most frequently involving the fingers and toes. Diltiazem (Cardizem) is a calcium channel blocker that relaxes smooth muscles of the arterioles by blocking the influx of calcium into the cells, thus reducing the frequency and severity of vasospastic attacks. Perfusion to the fingertips is improved and vasospastic attacks reduced. Diltiazem may decrease heart rate and blood pressure, but that is not the purpose in Raynaud's phenomenon. Skin turgor is most often a reflection of hydration status.

What is a priority nursing intervention in the care of a patient with a diagnosis of chronic venous insufficiency (CVI)? a. Application of topical antibiotics to venous ulcers b. Maintaining the patient's legs in a dependent position c. Administration of oral and/or subcutaneous anticoagulants d. Teaching the patient the correct use of compression stockings

*Correct Answer: D* CVI requires conscientious and consistent application of compression stockings. Anticoagulants are not necessarily indicated and antibiotics, if required, are typically oral or IV, not topical. The patient should avoid prolonged positioning with the limb in a dependent position.

The patient reports tenderness when she touches her leg over a vein. The nurse assesses warmth and a palpable cord in the area. The nurse knows the patient needs treatment to prevent which sequelae? a Pulmonary embolism b Pulmonary hypertension c Post-thrombotic syndrome d Venous thromboembolism

*Correct Answer: D* Rationale: The clinical manifestations are characteristic of a superficial vein thrombosis. If untreated, the clot may extend to deeper veins, and venous thromboembolism may occur. Pulmonary embolism, pulmonary hypertension, and post-thrombotic syndrome are the sequelae of venous thromboembolism.

A male patient was admitted for a possible ruptured aortic aneurysm, but had no back pain. Ten minutes later his assessment includes the following: sinus tachycardia at 138, BP palpable at 65 mm Hg, increasing waist circumference, and no urine output. How should the nurse interpret this assessment about the patient's aneurysm? a Tamponade will soon occur. b The renal arteries are involved. c Perfusion to the legs is impaired. d He is bleeding into the abdomen.

*Correct Answer: D* Rationale: The lack of back pain indicates the patient is most likely exsanguinating into the abdominal space, and the bleeding is likely to continue without surgical repair. A blockade of the blood flow will not occur in the abdominal space as it would in the retroperitoneal space where surrounding anatomic structures may control the bleeding. The lack of urine output does not indicate renal artery involvement, but that the bleeding is occurring above the renal arteries, which decreases the blood flow to the kidneys. There is no assessment data indicating decreased perfusion to the legs.

The nurse is preparing to administer a scheduled dose of enoxaparin (Lovenox) 30 mg subcutaneously. What should the nurse do to administer this medication correctly? a. Remove the air bubble in the prefilled syringe. b. Aspirate before injection to prevent IV administration. c. Rub the injection site after administration to enhance absorption. d. Pinch the skin between the thumb and forefinger before inserting the needle

*Correct Answer: D* Rationale: The nurse should gather together or "bunch up" the skin between the thumb and the forefinger before inserting the needle into the subcutaneous tissue. The nurse should not remove the air bubble in the prefilled syringe, aspirate, nor rub the site after injection.

The nurse is caring for a patient with a recent history of deep vein thrombosis (DVT). The patient now needs to undergo surgery for appendicitis. The nurse is reviewing the laboratory results for this patient before administering an ordered dose of vitamin K. The nurse determines that the medication is both safe to give and is most needed when the international normalized ratio (INR) is which result? a. 1.0 b. 1.2 c. 1.6 d. 2.2

*Correct Answer: D* Rationale: itamin K is the antidote to warfarin (Coumadin), which the patient has most likely been taking before admission for treatment of DVT. Warfarin is an anticoagulant that impairs the ability of the blood to clot. Therefore it is necessary to give vitamin K before surgery to reduce the risk of hemorrhage. The largest value of the INR indicates the greatest impairment of clotting ability, making 2.2 the correct selection.

A 39-year-old woman with a history of smoking and oral contraceptive use is admitted with a venous thromboembolism (VTE) and prescribed unfractionated heparin. What laboratory test should the nurse review to evaluate the expected effect of the heparin? a Platelet count b Activated clotting time (ACT) c International normalized ratio (INR) d Activated partial thromboplastin time (APTT)

*Correct Answer: D* Unfractionated heparin can be given by continuous IV for VTE treatment. When given IV, heparin requires frequent laboratory monitoring of clotting status as measured by activated partial thromboplastin time (aPTT). Platelet counts can decrease as an adverse reaction to heparin, but that is not the expected effect.

A nurse is caring for a patient with a diagnosis of deep venous thrombosis (DVT). The patient has an order to receive 30 mg enoxaparin (Lovenox). Which injection site should the nurse use to administer this medication safely? a. Buttock, upper outer quadrant b. Abdomen, anterior-lateral aspect c. Back of the arm, 2 inches away from a mole d. Anterolateral thigh, with no scar tissue nearby

*Correct Answer: b* Rationale: Enoxaparin (Lovenox) is a low-molecular-weight (LMW) heparin that is given as a deep subcutaneous injection in the right and left anterolateral abdomen. All subcutaneous injections should be given away from scars, lesions, or moles.

What medications should the nurse expect to include in the teaching plan to decrease the risk of cardiovascular events and death for PAD patients (select all that apply)? a Ramipril (Altace) b Cilostazol (Pletal) c Simvastatin (Zocor) d Clopidogrel (Plavix) e Warfarin (Coumadin) f Aspirin (acetylsalicylic acid)

*Correct Answers: a, c, d* Angiotensin-converting enzyme inhibitors (e.g., ramipril [Altace]) are used to control hypertension. Statins (e.g., simvastatin [Zocor]) are used for lipid management. Aspirin is used as an antiplatelet agent. Cilostazol (Pletal) is used for intermittent claudication, but it does not reduce CVD morbidity and mortality risks. Clopidogrel may be used if the patient cannot tolerate aspirin. Anticoagulants (e.g., warfarin [Coumadin]) are not recommended to prevent CVD events in PAD patients.

A key aspect of teaching for the patient on anticoagulant therapy includes which instructions? a. Monitor for and report any signs of bleeding. b. Do not take acetaminophen (Tylenol) for a headache. c. Decrease your dietary intake of foods containing vitamin K. d. Arrange to have blood drawn routinely to check drug levels.

*Correct answer: a* Rationale: Patients taking anticoagulants should be taught to monitor and report any signs of bleeding, which can be a serious complication. Other important patient teaching includes maintenance of a consistent intake of foods containing vitamin K, avoidance of supplements that contain vitamin K, and routine coagulation laboratory studies if a patient is taking warfarin.

A patient is admitted to the hospital with a diagnosis of abdominal aortic aneurysm. Which signs and symptoms would suggest that his aneurysm has ruptured? a. Sudden shortness of breath and hemoptysis b. Sudden, severe low back pain and bruising along his flank c. Gradually increasing substernal chest pain and diaphoresis d. Sudden, patchy blue mottling on feet and toes and rest pain

*Correct answer: b* Rationale: The clinical manifestations of a ruptured abdominal aortic aneurysm include severe back pain, back or flank ecchymosis (Grey Turner's sign), and hypovolemic shock (tachycardia, hypotension, pale clammy skin, decreased urine output, altered level of consciousness, and abdominal tenderness).

The patient at highest risk for venous thromboembolism (VTE) is a. a 62-year-old man with spider veins who is having arthroscopic knee surgery. b. a 32-year-old woman who smokes, takes oral contraceptives, and is planning a trip to Europe. c. a 26-year-old woman who is 3 days postpartum and received maintenance IV fluids for 12 hours during her labor. d. an active 72-year-old man at home recovering from transurethral resection of the prostate for benign prostatic hyperplasia.

*Correct answer: b* Rationale: Three important factors (called Virchow's triad) in the etiology of venous thrombosis are (1) venous stasis, (2) damage of the endothelium (inner lining of the vein), and (3) hypercoagulability of the blood. Patients at risk for venous thrombosis usually have predisposing conditions for these three disorders (see Table 38-8). The 32-year-old woman has the highest risk: long trips without adequate exercise (venous stasis), tobacco use, and use of oral contraceptives. Note: The likelihood of hypercoagulability of blood is increased in women older than 35 years who use tobacco.

Rest pain is a manifestation of PAD that occurs due to a chronic a. vasospasm of small cutaneous arteries in the feet. b. increase in retrograde venous blood flow in the legs. c. decrease in arterial blood flow to the nerves of the feet. d. decrease in arterial blood flow to the leg muscles during exercise.

*Correct answer: c* Rationale: Rest pain most often occurs in the forefoot or toes and is aggravated by limb elevation. Rest pain occurs when blood flow is insufficient to meet basic metabolic requirements of the distal tissues. Rest pain occurs more often at night because cardiac output tends to drop during sleep and the limbs are at the level of the heart. Patients often try to achieve partial pain relief by dangling the leg over the side of the bed or sleeping in a chair to allow gravity to maximize blood flow.

A 50-year-old woman weighs 95 kg and has a history of tobacco use, high blood pressure, high sodium intake, and sedentary lifestyle. When developing an individualized care plan for her, the nurse determines that the most important risk factors for peripheral artery disease (PAD) that need to be modified are: a. weight and diet. b. activity level and diet. c. tobacco use and high blood pressure. d. sedentary lifestyle and high blood pressure.

*Correct answer: c* Rationale: Significant risk factors for peripheral artery disease include tobacco use, hyperlipidemia, elevated levels of high-sensitivity C-reactive protein, diabetes mellitus, and uncontrolled hypertension; the most important is tobacco use. Other risk factors include family history, hypertriglyceridemia, hyperuricemia, increasing age, obesity, sedentary lifestyle, and stress.

A patient with infective endocarditis develops sudden left leg pain with pallor, paresthesia, and a loss of peripheral pulses. The nurse's initial action should be to a. elevate the leg to promote venous return. b. start anticoagulant therapy with IV heparin. c. notify the physician of the change in peripheral perfusion. d. place the bed in reverse Trendelenburg to promote perfusion.

*Correct answer: c* Rationale: The patient has potentially developed acute arterial ischemia (sudden interruption in the arterial blood supply to the extremity), caused by an embolism from a cardiac thrombus that occurred as a complication of infective endocarditis. Clinical manifestations of acute arterial ischemia include any or all of the six Ps : pain, pallor, paralysis, pulselessness, paresthesia, and poikilothermia. Without immediate intervention, ischemia may progress quickly to tissue necrosis and gangrene within a few hours. If the nurse detects these signs, the physician should be notified immediately.

Priority nursing measures after an abdominal aortic aneurysm repair include a. assessment of cranial nerves and mental status. b. administration of IV heparin and monitoring of aPTT. c. administration of IV fluids and monitoring of kidney function. d. elevation of the legs and application of elastic compression stockings

*Correct answer: c* Rationale: Postoperative priorities include administration of IV fluids and maintenance of renal perfusion. An adequate blood pressure is important for maintaining graft patency, and administration of IV fluids and blood components (as indicated) is essential for adequate blood flow. The nurse should evaluate renal function by measuring hourly urine output and monitoring daily blood urea nitrogen (BUN) and serum creatinine levels. Irreversible renal failure may occur after aortic surgery, particularly in individuals at high risk.

In planning care and patient teaching for the patient with venous leg ulcers, the nurse recognizes that the most important intervention in healing and control of this condition is a. sclerotherapy. b. using moist environment dressings. c. taking horse chestnut seed extract daily. d. applying elastic compression stockings.

*Correct answer: d* Rationale: Compression is essential for treating chronic venous insufficiency (CVI), healing venous ulcers, and preventing ulcer recurrence. Use of custom-fitted elastic compression stockings is one option for compression therapy.

The recommended treatment for an initial VTE in an otherwise healthy person with no significant co-morbidities would include a. IV argatroban (Acova) as an inpatient. b. IV unfractionated heparin as an inpatient. c. subcutaneous unfractionated heparin as an outpatient. d. subcutaneous low-molecular-weight heparin as an outpatient.

*Correct answer: d* Rationale: Patients with confirmed VTE should receive initial treatment with low-molecular-weight heparin (LMWH), unfractionated heparin (UH), fondaparinux, or rivaroxaban, followed by warfarin for 3 months to maintain the international normalized ratio (INR) between 2.0 and 3.0 for 24 hours. Patients with multiple comorbid conditions, complex medical issues, or a very large VTE usually are hospitalized for treatment and typically receive intravenous UH. LMWH only for 3 months is another option for patients with acute VTE. Depending on the clinical presentation, patients often can be managed safely and effectively as outpatients.

Which clinical manifestations are seen in patients with either Buerger's disease or Raynaud's phenomenon (select all that apply)? a. Intermittent fevers b. Sensitivity to cold temperatures c. Gangrenous ulcers on fingertips d. Color changes of fingers and toes e. Episodes of superficial vein thrombosis

*Correct answers: b, c, d* Rationale: Both Buerger's disease and Raynaud's phenomenon have the following clinical manifestations in common: cold sensitivity, ischemic and gangrenous ulcers on fingertips, and color changes of the distal extremity (fingers or toes).

Which are probable clinical findings in a person with an acute lower extremity VTE (select all that apply)? a. Pallor and coolness of foot and calf b. Mild to moderate calf pain and tenderness c. Grossly diminished or absent pedal pulses d. Unilateral edema and induration of the thigh e. Palpable cord along a superficial varicose vein

*Correct answers: b, d* Rationale: The patient with lower extremity venous thromboembolism (VTE) may or may not have unilateral leg edema, extremity pain, a sense of fullness in the thigh or calf, paresthesias, warm skin, erythema, or a systemic temperature greater than 100.4 F (38 C). If the calf is involved, it may be tender to palpation.

The nurse is reviewing the instillation technique for both eye ointment and eye drops with the parent of a pediatric client diagnosed with bacterial conjunctivitis. Which statement made by the parent would indicate that learning has taken place? 1. "I will be careful not to touch the eye or eyelid during administration." 2. "I will place my child on the left side to administer drops in the right eye." 3. "I will administer the eye ointment and then wait 5 minutes and administer the eye drops." 4. "I will have my child blink after the instillation to encourage thorough distribution of the eye drops."

1. "I will be careful not to touch the eye or eyelid during administration."

A client who is scheduled for cataract surgery requires preoperative instillation of cyclopentolate eye drops as prescribed. The client asks the nurse why this medication is needed, and the nurse provides education. Which statement by the client indicates that teaching has been effective? 1. "The medication dilates the pupil of the operative eye." 2. "The medication constricts the pupil of the operative eye." 3. "The medication is needed for the initiation of miosis in the operative eye." 4. "The medication provides the necessary lubrication to the nonoperative eye."

1. "The medication dilates the pupil of the operative eye."

A client is prescribed an eye drop and an eye ointment for the right eye. How should the nurse best administer the medications? 1. Administer the eye drop first, followed by the eye ointment. 2. Administer the eye ointment first, followed by the eye drop. 3. Administer the eye drop, wait 15 minutes, and administer the eye ointment. 4. Administer the eye ointment, wait 15 minutes, and administer the eye drop.

1. Administer the eye drop first, followed by the eye ointment.

The home care nurse is reviewing the record of a client newly diagnosed with glaucoma who is scheduled for a home visit. The nurse notes that the health care provider (HCP) has prescribed atropine sulfate and pilocarpine hydrochloride eye drops. The nurse should contact the HCP before the home visit for which reason? 1. Clarify the prescription for the atropine sulfate. 2. Clarify the prescription for the pilocarpine hydrochloride. 3. Determine the date of the scheduled follow-up HCP visit. 4. Determine the extent of the intraocular pressure caused by the glaucoma.

1. Clarify the prescription for the atropine sulfate.

A client being discharged to home with a prescription for eye drops to be given in the left eye has received instructions regarding self-administration of the drops. The nurse determines that the client needs further instruction if, on return demonstration, the client takes which action? 1. Lies supine, pulls up on the upper lid, and puts the drop in the upper lid 2. Lies supine, pulls down on the lower lid, and puts the drop in the lower lid 3. Tilts the head back, pulls down on the lower lid, and puts the drop in the lower lid 4. Lies with head to the right, puts the drop in the inner canthus, and slowly turns to the left while blinking

1. Lies supine, pulls up on the upper lid, and puts the drop in the upper lid

Betaxolol eye drops have been prescribed for a client with glaucoma. The home health nurse preparing to visit the client develops a plan of care that includes monitoring for the side/adverse effects of this medication by taking which assessment action? 1. Monitoring body weight 2. Assessing the glucose level 3. Assessing peripheral pulses 4. Monitoring body temperature

1. Monitoring body weight

Betaxolol eye drops have been prescribed for a client with glaucoma. The nurse monitoring this client for side/adverse effects of the medication would place highest priority on which assessment? 1. Pulse rate 2. Blood glucose 3. Respiratory rate 4. Oxygen saturation

1. Pulse rate

The nurse prepares a client for ear irrigation as prescribed by the health care provider. Which action should the nurse take when performing the procedure? 1. Warm the irrigating solution to 98.6°F (37.0°C) . 2. Position the client with the affected side up following the irrigation. 3. Direct a slow, steady stream of irrigation solution toward the eardrum. 4. Assist the client to turn his or her head so that the ear to be irrigated is facing upward.

1. Warm the irrigating solution to 98.6°F (37.0°C) .

The nurse is preparing to administer eye drops. Which interventions should the nurse take to administer the drops? Select all that apply. 1. Wash hands. 2. Put gloves on. 3. Place the drop in the conjunctival sac. 4. Pull the lower lid down against the cheekbone. 5. Instruct the client to squeeze the eyes shut after instilling the eye drop. 6. Instruct the client to tilt the head forward, open the eyes, and look down.

1. Wash hands. 2. Put gloves on. 3. Place the drop in the conjunctival sac. 4. Pull the lower lid down against the cheekbone.

The client receives timolol (Timoptic) eyedrops for glaucoma. The nurse has completed medication education and evaluates learning has occurred when the client makes which statement? 1. "I will discuss use of over-the-counter (OTC) medications with my physician." 2. "I will store my drops in the refrigerator to keep them fresh." 3. "I will restrict my caffeine to one cup of coffee per day." 4. "I will discontinue the drops if my eye looks red."

1. "I will discuss use of over-the-counter (OTC) medications with my physician."

Which statements would the nurse interpret as indicating that the client understands the diagnosis of open-angle glaucoma? Select all that apply. 1. "My eye cannot drain the fluid that it produces." 2. "One of the first signs of this is pain behind my eyes." 3. "My type of glaucoma is a medical emergency." 4. "This is a gradual process; it may take years to develop." 5. "This is the most common kind of glaucoma."

1. "My eye cannot drain the fluid that it produces." 4. "This is a gradual process; it may take years to develop." 5. "This is the most common kind of glaucoma."

The client receives eyedrops as treatment for glaucoma. The client calls the clinic one day and tells the nurse that his eye color is changing. What is the best response by the nurse? 1. "This is an expected side effect of the medication." 2. "This is unusual; please come in for an evaluation." 3. "Are you sure that your eyes have changed color?" 4. "What do you mean that your eyes have changed color?"

1. "This is an expected side effect of the medication."

10. The nurse is assessing a client who is complaining of visual impairment. The nurse knows that which of the following risk factors are associated with development of cataracts? Cigarette smoking_1___ Diabetes Insipidus_2___ Alcohol consumption___3_ High Blood pressure__4__ Migraine headaches__5__ Exposure to ultraviolet light_6___ Diet high in fat___7_

1346

When caring for a patient who has started anticoagulant therapy with warfarin (Coumadin), the nurse knows not to expect therapeutic benefits for: a. At least 12 hours b. The first 24 hours c. 2-3 days d. 1 week

2-3 days

The nurse would question the health care provider if which medication were prescribed for a client with glaucoma? 1. Carbachol 2. Atropine sulfate 3. Pilocarpine nitrate 4. Pilocarpine hydrochloride

2. Atropine sulfate

Pilocarpine hydrochloride is prescribed for a client with glaucoma. The nurse checks the medication supply room to ensure that atropine sulfate is available for administration in the event that systemic toxicity occurs from the use of pilocarpine hydrochloride. The nurse also monitors for which sign of systemic toxicity? 1. Anorexia 2. Bradycardia 3. Tachycardia 4. Hypertension

2. Bradycardia

A client with chronic glaucoma is being started on medication therapy with acetazolamide. The nurse teaches the client that which can occur early with the use of this medication? 1. Fatigue 2. Diuresis 3. Headache 4. Loss of libido

2. Diuresis

The client tells the nurse that when he uses his timolol (Timoptic) eyedrops, they sting his eyes. What is the best response by the nurse? 1. "Hold the next dose and contact your physician." 2. "This is a normal and expected effect of the drops." 3. "Your eyedrops may have expired; check the date." 4. "You should wash your eyes immediately with saline."

2. "This is a normal and expected effect of the drops."

The client tells the nurse that he experiences frequent eye irritation even after using over-the-counter (OTC) medications. What is the best recommendation by the nurse? 1. Increase your fluid intake; you are probably dehydrated. 2. See your eye doctor for further evaluation. 3. This sounds like an allergic response; try an antihistamine. 4. Use normal saline rinses instead of over-the-counter (OTC) preparations

2. See your eye doctor for further evaluation.

A client has been diagnosed with closed-angle glaucoma. Which findings would the nurse interpret as indicating that the pharmacologic treatment regimen has been successful? Select all that apply. 1. The client reports being hungry. 2. The client's last two intraocular pressure readings have been 19 mmHg and 18 mmHg. 3. The client's eye is not as bloodshot. 4. The client reports a sensation of drainage down the nose. 5. The client's cough has diminished.

2. The client's last two intraocular pressure readings have been 19 mmHg and 18 mmHg. 3. The client's eye is not as bloodshot

What should the nurse do for a client who just had cataract surgery? 1 Instruct the client to avoid driving for several weeks. 2 Teach the client coughing and deep-breathing techniques. 3 Advise the client to refrain from vigorous brushing of the teeth and hair. 4 Encourage the client to perform eye exercises to strengthen the ocular musculature.

3 Activities such as rigorous brushing of hair and teeth cause increased intraocular pressure and may lead to hemorrhage in the anterior chamber. 1 This is unnecessary; clients are usually permitted to drive before this time. 2 Coughing and deep breathing can increase intraocular pressure and should be avoided. 4 Weakening of the eye musculature is not related to cataracts.

A client with glaucoma asks a nurse about future treatment and precautions. What information should the nurse's explanation include? 1 Avoidance of cholinergics 2 Surgical replacement of lens 3 Continuation of therapy for life 4 Prevention of high blood pressure

3 Therapy must be continued for life to prevent damage to the optic nerve from increased intraocular pressure. 1 These are used in the treatment of glaucoma; anticholinergics are contraindicated. 2 This is the treatment for cataracts. 4 There is an increase in intraocular pressure with glaucoma; the blood pressure may be unaffected.

normal pupil size

3-5 mm

The preoperative medication sheet identifies that cyclopentolate is prescribed for a client before cataract surgery. The client asks the nurse what the medication is for, and the nurse provides education. Which statement by the client indicates that teaching has been effective? 1. "It lubricates the eye." 2. "It makes my pupils smaller." 3. "It paralyzes the muscles in my eye." 4. "It causes me vessels to become smaller."

3. "It paralyzes the muscles in my eye."

A miotic medication has been prescribed for the client with glaucoma and the client asks the nurse about the purpose of the medication. Which response should the nurse provide to the client? 1. "The medication will help dilate the eye to prevent pressure from occurring." 2. "The medication will relax the muscles of the eyes and prevent blurred vision." 3. "The medication causes the pupil to constrict and will lower the pressure in the eye." 4. "The medication will help block the responses that are sent to the muscles in the eye."

3. "The medication causes the pupil to constrict and will lower the pressure in the eye."

The nursing student is assigned to care for a client with glaucoma for whom pilocarpine hydrochloride eye drops have been prescribed. The nursing instructor asks the student to describe the action of the eye medication. Which statement by the student indicates an understanding of the purpose of this medication? 1. "The medication prevents blurred vision by relaxing the muscles of the eyes." 2. "The medication dilates the eye to prevent increased pressure from occurring." 3. "The medication increases the blood flow to the retina and also will lower the pressure in the eye." 4. "The medication blocks responses that are sent to the brain that direct the actions of the muscles in the eye."

3. "The medication increases the blood flow to the retina and also will lower the pressure in the eye."

A client with glaucoma is receiving acetazolamide. The nurse educator provides education to a group of nurses about the indications for and effect of this medication. Which statement by one of the nurses indicates that the teaching has been effective? 1. "This works to prevent hypertension." 2. "This works to prevent hyperthermia." 3. "This works to decrease intraocular pressure." 4. "This works to maintain an adequate blood pressure for cerebral perfusion."

3. "This works to decrease intraocular pressure."

A client was just admitted to the hospital to rule out a gastrointestinal (GI) bleed. The client has brought several bottles of medications prescribed by different specialists. During the admission assessment, the client states, "Lately, I have been hearing some roaring sounds in my ears, especially when I am alone." Which medication would the nurse identify as the cause of the client's complaint? 1. Doxycycline 2. Atropine sulfate 3. Acetylsalicylic acid 4. Diltiazem hydrochloride

3. Acetylsalicylic acid

The nurse working in a long-term care facility notes that several clients are taking pilocarpine hydrochloride eye drops. The nurse ensures that which medication is available on the nursing unit for use if a client should develop systemic toxicity from pilocarpine hydrochloride? 1. Disulfiram 2. Cyclopentolate 3. Atropine sulfate 4. Naloxone hydrochloride

3. Atropine sulfate

In preparation for cataract surgery, the nurse is to administer cyclopentolate eye drops at 0900 for surgery that is scheduled for 0915. What initial action should the nurse take in relation to the characteristics of the medication action? 1. Provide lubrication to the operative eye prior to giving the eye drops. 2. Call the surgeon, as this medication will further constrict the operative pupil. 3. Consult the surgeon, as there is not sufficient time for the dilative effects to occur. 4. Give the medication as prescribed; the surgeon needs optimal constriction of the pupil.

3. Consult the surgeon, as there is not sufficient time for the dilative effects to occur.

Betaxolol hydrochloride eye drops have been prescribed for a client with glaucoma. Which nursing action is most appropriate related to monitoring for side and adverse effects of this medication? 1. Assessing for edema 2. Monitoring temperature 3. Monitoring blood pressure 4. Assessing blood glucose level

3. Monitoring blood pressure

Ear drops are prescribed for an infant with otitis media. Which is the most appropriate method to administer ear drops to an infant? 1. Pull up and back on the pinna, and direct the solution onto the eardrum. 2. Pull down and back on the pinna, and direct the solution onto the eardrum. 3. Pull down and back on the pinna, and direct the solution toward the wall of the canal. 4. Pull up and back on the pinna, and direct the solution toward the wall of the canal.

3. Pull down and back on the pinna, and direct the solution toward the wall of the canal.

The client receives latanoprost (Xalatan) and another eyedrop. The nurse has completed medication education and evaluates that learning has occurred when the client makes which statement? 1. "I will give two drops of one medication, wait one minute, and then give two drops of the other medication." 2. "I should lay the dropper against my eyelid for stability when putting in drops." 3. "I will remove my contact lens before instilling these eyedrops." 4. "I may notice that my eyelashes get thinner while I am using this medication."

3. "I will remove my contact lens before instilling these eyedrops."

Which desired effect of therapy should the nurse explain to the client who has primary angle-closure glaucoma? 1 Dilating the pupil 2 Resting the eye muscles 3 Preventing secondary infection 4 Controlling intraocular pressure

4 Glaucoma is a disease in which there is increased intraocular pressure resulting from narrowing of the aqueous outflow channel (canal of Schlemm). This can lead to blindness, caused by compression of the nutritive blood vessels supplying the rods and cones. 1 Pupil dilation increases intraocular pressure because it narrows the canal of Schlemm. 2 Intraocular pressure is not affected by activity of the eye. 3 Although secondary infections are not desirable, the priority is to maintain vision by controlling the pressure.

Which clinical indicator is the nurse most likely to identify when exploring the history of a client with open-angle glaucoma? 1 Constant blurring 2 Abrupt attacks of acute pain 3 Sudden, complete loss of vision 4 Impairment of peripheral vision

4 Open-angle glaucoma has an insidious onset, with increased intraocular pressure causing pressure on the retina and blood vessels in the eye. Peripheral vision is decreased as the visual field progressively diminishes. 1 This may occur with untreated acute angle-closure glaucoma. 2 Pain occurs in acute angle-closure, not open-angle, glaucoma. 3 Occlusions of the central retinal artery or retinal detachment will cause a sudden loss of vision

A nurse is caring for a client who is scheduled for surgery for a detached retina. Which goal of surgery identified by the client indicates that the preoperative teaching was effective? 1 Promote growth of new retinal cells 2 Adhere the sclera to the choroid layer 3 Graft a healthy piece of retina in place 4 Create a scar that aids in healing retinal holes

4 Scar formation seals the hole and promotes attachment of the two retinal surfaces. 1 The retina is part of the nervous system; it does not regenerate or grow new cells. 2 The sclera is not involved; the retina adjoins and is nourished by the choroid. 3 This is not the treatment used; treatment includes the formation of a scar by the use of lasers or surgical "buckling."

In preparation for cataract surgery, the nurse is to administer prescribed eye drops. The nurse reviews the health care provider's prescriptions, expecting which type of eye drops to be prescribed? 1. A miotic agent 2. A thiazide diuretic 3. An osmotic diuretic 4. A mydriatic medication

4. A mydriatic medication

Which medication, if prescribed for the client with glaucoma, should the nurse question? 1. Betaxolol 2. Pilocarpine 3. Erythromycin 4. Atropine sulfate

4. Atropine sulfate

A client has been prescribed betaxolol eye drops for the treatment of glaucoma. The ambulatory care nurse determines that the client understands proper medication use if the client states the need to return to the office for monitoring of what item(s)? 1. Hearing acuity 2. Blood glucose level 3. Presence of calf pain 4. Blood pressure and apical pulse

4. Blood pressure and apical pulse

The nurse is performing an admission assessment on a client who has a history of glaucoma and uses latanoprost eye drops. Which assessment finding would indicate a side/adverse effect of these eye drops? 1. Irregular pulse 2. Periorbital edema 3. Elevated blood pressure 4. Brown pigmentation of the iris

4. Brown pigmentation of the iris

A client with glaucoma is given a prescription for a pilocarpine ocular system. The nurse plans to provide which instruction to the client on how to use the medication? 1. Apply ½ inch into the eye at bedtime. 2. Apply one drop of the solution four times a day. 3. Remove and replace the ocular system every 48 hours. 4. Check the eye each morning to make sure that the system is in place.

4. Check the eye each morning to make sure that the system is in place.

The nurse teaching a mother how to administer ear drops to an infant tells the mother to pull the child's ear in which direction? 1. Up and back and direct the solution onto the eardrum 2. Down and forward and direct the solution onto the eardrum 3. Up and forward and direct the solution toward the wall of the canal 4. Down and back and direct the solution toward the wall of the canal

4. Down and back and direct the solution toward the wall of the canal

When teaching a client with glaucoma about the effects of a miotic medication, the nurse should tell the client that the medication will produce which effect? 1. Reshape the lens to eliminate blurred vision 2. Dilate the pupil to reduce intraocular pressure 3. Interrupt the drainage of aqueous humor from the eye 4. Lower intraocular pressure and improve blood flow to the retina

4. Lower intraocular pressure and improve blood flow to the retina

The nurse is providing instructions to a client who will be self-administering eye drops. To minimize systemic absorption of the eye drops, the nurse should instruct the client to take which action? 1. Eat before instilling the drops. 2. Swallow several times after instilling the drops. 3. Blink vigorously to encourage tearing after instilling the drops. 4. Occlude the nasolacrimal duct with a finger after instilling the drops.

4. Occlude the nasolacrimal duct with a finger after instilling the drops.

The nurse is teaching a mother to instill drops in her infant's ear. The nurse explains that to give the ear drops correctly, the mother needs to take which action? 1. Pull up and back on the earlobe and direct the solution toward the eardrum. 2. Pull down and back on the auricle and direct the solution toward the eardrum. 3. Pull up and back on the auricle and direct the solution toward the wall of the canal. 4. Pull down and back on the earlobe and direct the solution toward the wall of the canal.

4. Pull down and back on the earlobe and direct the solution toward the wall of the canal.

The nurse has taught the client about open-angle glaucoma. The nurse evaluates learning has occurred when the client makes which statement? 1. "The eyedrops only need to be used when my eyes hurt." 2. "I will need to continually increase the dose of my eyedrops." 3. "I can stop the eyedrops when the glaucoma has resolved." 4. "I must use my eyedrops as prescribed for the rest of my life."

4. "I must use my eyedrops as prescribed for the rest of my life."

The client is scheduled for an eye exam. Prior to the exam the physician will put a cycloplegic drug, atropine sulfate (Isopto Atropine) eyedrops in the client's eyes. What will the nurse teach the client about these eyedrops? 1. The drugs will dilate the pupil and lubricate the eye to provide additional comfort during the examination. 2. The drops will paralyze the muscles that move the eye so that examination can take place. 3. The drops will dilate the pupil so that the physician can better visualize the retina during examination. 4. The drops dilate the pupil and paralyze the ciliary muscle to prevent the lens from moving during examination.

4. The drops dilate the pupil and paralyze the ciliary muscle to prevent the lens from moving during examination.

The primary mechanism of action of beta-adrenergic blockers in the treatment of open-angle glaucoma is to 1. increase the outflow of aqueous humor. 2. constrict the pupil. 3. dilate the pupil to increase outflow. 4. reduce production of aqueous humor.

4. reduce production of aqueous humor.

1. Call the health care provider (HCP).

741. During the early postoperative period, a client who has undergone a cataract extraction complains of nausea and severe eye pain over the operative site. What should be the initial nursing action

3. Eye medications will need to be administered for life.

742. The nurse is developing a teaching plan for a client with glaucoma. Which instruction should the nurse include in the plan of care?

4. A sense of a curtain falling across the field of vision

743. The nurse is performing an admission assessment on a client with a diagnosis of detached retina. Which sign or symptom is associated with this eye disorder?

4. A red, dull, thick, and immobile tympanic membrane

744. The nurse is performing an otoscopic examination on a client with mastoiditis. On examination of the tympanic membrane, which finding should the nurse expect to observe?

2. Tinnitus

745. A client is diagnosed with a disorder involving the inner ear. Which is the most common client complaint associated with a disorder involving this part of the ear?

4. Blurred vision

746. The nurse is performing an assessment on a client with a suspected diagnosis of cataract. Which clinical manifestation should the nurse expect to note in the early stages of cataract formation?

2. A semi-Fowler's position

747. A client arrives in the emergency department following an automobile crash. The client's forehead hit the steering wheel and a hyphema is diagnosed. The nurse should place the client in which position?

1. Apply ice to the affected eye.

748. The client sustains a contusion of the eyeball following a traumatic injury with a blunt object. Which intervention should be initiated immediately?

2. Perform visual acuity tests.

749. A client arrives in the emergency department with a penetrating eye injury from wood chips that occurred while cutting wood. The nurse assesses the eye and notes a piece of wood protruding from the eye. What is the initial nursing action?

1. Irrigate the eyes with water.

751. A woman was working in her garden. She accidentally sprayed insecticide into her right eye. She calls the emergency department, frantic and screaming for help. The nurse should instruct the woman to take which immediate action?

1. Avoid activities that require bending over. 3. Take acetaminophen for minor eye discomfort. 5. Place an eye shield on the surgical eye at bedtime. 6. Contact the surgeon if a decrease in visual acuity occurs.

752. The nurse is preparing a teaching plan for a client who had a cataract extraction with intraocular implantation. Which home care measures should the nurse include in the plan? Select all that apply.

2. Note the time of day the test was done.

753. Tonometry is performed on a client with a suspected diagnosis of glaucoma. The nurse looks at the documented test results and notes an intraocular pressure (IOP) value of 23. What should be the nurse's initial action?

4. Cranial nerve VII, facial nerve

754. The nurse is caring for a client following craniotomy for removal of an acoustic neuroma. Assessment of which cranial nerve would identify a complication specifically associated with this surgery?

3. Speak at normal tone and pitch, slowly and clearly.

755. The nurse notes that the health care provider has documented a diagnosis of presbycusis on a client's chart. Based on this information, what action should the nurse take?

2. Avoid sudden head movements.

756. A client with Meniere's disease is experiencing severe vertigo. Which instruction should the nurse give to the client to assist in controlling the vertigo?

1. The right eye is tested, followed by the left eye, and then both eyes are tested.

757. The nurse is preparing to test the visual acuity of a client, using a Snellen chart. Which identifies the accurate procedure for this visual acuity test?

2. Instruct the client that he or she may need glasses when driving.

758. A client's vision is tested with a Snellen chart. The results of the tests are documented as 20/60. What action should the nurse implement based on this finding?

3. Speak at a normal volume.

759. The nurse is caring for a hearing-impaired client. Which approach will facilitate communication?

A client diagnosed with atherosclerosis has been prescribed lovastatin (Mevacor). Which statement by the client indicates a need for further teaching? A. "I won't need to change my diet because now I'm taking a pill." B. "I'll follow up with my nurse practitioner on a regular basis." C. "I need to quit smoking as soon as I possibly can." D. "I shouldn't drink grapefruit juice while on this drug."

A

During an assessment of a patient's abdomen, a pulsating abdominal mass is noted by the healthcare provider. Which of the following should be the healthcare provider's next action? A. Assess femoral pulses B. Obtain a bladder scan C. Measure the abdominal circumference D. Ask the patient to perform a Valsalva maneuver

A

Essential feature of glaucoma is: A. Optic neuropathy B. Raised intraocular pressure C. Reduced vision D. Painful eye

A

The client who has tinnitus is taking these drugs daily: 1 multiple vitamin, losartan (Cozaar) 50 mg, aspirin 650 mg, and diphenhydramine (Benadryl) 25 mg. Which drug alerts the nurse to a possible cause of tinnitus? A. Aspirin B. Losartan C. Multiple vitamin D. Diphenhydramine

A

The nurse is caring for a client with chronic venous stasis ulcers. Which statement by the client indicates a need for further health teaching? A. "I'll wear compression stockings at night." B. "I'll keep my affected leg above my heart." C. "I'll eat protein and vitamin C foods to help heal the ulcer" D. "I'll change my dressing every 3 to 5 days as needed."

A

Which lifestyle modification does the nurse suggest to the client with Ménière's disease to reduce the frequency or intensity of acute episodes? A. Quitting cigarette smoking B. Avoiding aspirin-containing drugs C. Reducing the amount of saturated fats in the diet D. Avoiding crowds and people who have upper respiratory infections

A

Which precaution is most important for the nurse to teach a 62-year- old client newly diagnosed with early-stage dry age-related macular degeneration? A. Quit smoking B. Quit drinking alcoholic beverages C. Eat more dark green, red, and yellow vegetables D. Wear dark glasses whenever he or she is outside or in bright interior lighting environments

A

The nurse is caring for a client with peripheral arterial disease (PAD). For which symptoms does the nurse assess? A. Reproducible leg pain with exercise B. Unilateral swelling of affected leg C. Decreased pain when legs are elevated D. Pulse oximetry reading of 90%

A Claudication (leg pain with ambulation due to ischemia) is reproducible in similar circumstances. Unilateral swelling is typical of venous problems such as deep vein thrombosis. With PAD, pain decreases with legs in the dependent position. Pulse oximetry readings reflect the amount of oxygen bound to hemoglobin; PAD results from atherosclerotic occlusion of peripheral arteries.

A client has just undergone arterial revascularization. Which statement by the client indicates a need for further teaching related to postoperative care? A. "My leg might turn very white after the surgery." B. "I should be concerned if my foot turns blue." C. "I should report a fever or any drainage." D. "Warmness, redness, and swelling are expected."

A Pallor is one of the signs of decreased perfusion along with increased pain, poikilothermia, paresthesia, pulselessness, and paralysis

The nurse is caring for a client following enucleation. The nurse notes the presence of bright red blood drainage on the dressing. Which nursing action is appropriate? A. Notify the physician B. Continue to monitor the drainage C. Document the finding D. Mark the drainage on the dressing and monitor for any increase in bleeding.

A If the nurse notes the presence of bright red drainage on the dressing, it must be reported to the physician because this indicated hemorrhage.

The nurse is assigned to all of these clients. Which client should be assessed first? A. The client who had percutaneous transluminal angioplasty (PTA) of the right femoral artery 30 minutes ago B. The client admitted with hypertensive crisis who has a nitroprusside (Nipride) drip and blood pressure of 149/80 mm Hg C. The client with peripheral vascular disease who has a left leg ulcer draining purulent yellow fluid D. The client who had a right femoral-popliteal bypass 3 days ago and has ongoing edema of the foot

A The client who had PTA should have checks of vascular status and vital signs every 15 minutes in the first hour after the procedure.

The nurse has notes that the physician has a diagnosis of presbycusis on the client's chart. The nurse plans care knowing the condition is: A. A sensorineural hearing loss that occurs with aging B. A conductive hearing loss that occurs with aging. C. Tinnitus that occurs with aging D. Nystagmus that occurs with aging

A Presbycusis is a type of hearing loss that occurs with aging. Presbycusis is a gradual sensorineural loss caused by nerve degeneration in the inner ear or auditory nerve.

For a client having an episode of acute narrow-angle glaucoma, a nurse expects to give which of the following medications? A. Acetazolamide (Diamox) B. Atropine C. Furosemide (Lasix) D. Urokinase (Abbokinase

A Acetazolamide, a carbonic anhydrase inhibitor, decreases intraocular pressure (IOP) by decreasing the secretion of aqueous humor. Atropine dilates the pupil and decreases outflow of aqueous humor, causing further increase in IOP. Lasix is a loop diuretic, and Urokinase is a thrombolytic agent; they aren't used for the treatment of glaucoma.

Cataract surgery results in aphakia. Which of the following statements best describes this term? A. Absence of the crystalline lens B. A "keyhole" pupil C. Loss of accommodation D. Retinal detachment

A Aphakia means without lens, a keyhole pupil results from iridectomy. Loss of accommodation is a normal response to aging. A retinal detachment is usually associated with retinal holes created by vitreous traction.

For a client complaining of periocular aching after a surgical repair of a detached retina, which medication would be the most appropriate analgesic? A. Acetaminophen B. Codeine C. Meperidine D. Morphine

A Because the discomfort is typically mild after surgery to repair a detached retina, a mild analgesic such as acetaminophen would be used. Codeine is constipating and may lead to straining and increased intraocular pressure (IOP). Meperidine often causes nausea and vomiting, further adding to the client's level of discomfort, and vomiting may lead to increased IOP. Morphine causes nausea, vomiting, and constipation, which should be avoided after surgery.

Mang Isko, a 68-year-old widower, has been stricken with cataracts about year ago. Which assessment date would the nurse expect when collecting the nursing history from the client? A. Blurred vision B. Eye pain C. Floaters D. Eye redness

A Cataracts lead to progressive worsening and blurring of vision. Eye pain and redness, common with glaucoma, are not present with cataracts. Floaters are characteristics of retinal detachment.

The nurse is caring for a client with a diagnosis of detached retina. Which assessment sign would indicate that bleeding has occurred as a result of the retinal detachment? A. Complaints of a burst of black spots or floaters B. A sudden sharp pain in the eye C. Total loss of vision D. A reddened conjunctiva

A Complaints of a sudden burst of black spots or floaters indicate that bleeding has occurred as a result of the detachment.

During the early postoperative period, the client who had a cataract extraction complains of nausea and severe eye pain over the operative site. The initial nursing action is to: A. Call the physician B. Administer the ordered main medication and antiemetic C. Reassure the client that this is normal. D. Turn the client on his or her operative side

A Severe pain or pain accompanied by nausea is an indicator of increased intraocular pressure and should be reported to the physician immediately. The other options are inappropriate.

Which of the following symptoms would occur in a client with a detached retina? A. Flashing lights and floaters B. Homonymous hemianopia C Loss of central vision D. Ptosis

A Signs and symptoms of retinal detachment include abrupt flashing lights, floaters, loss of peripheral vision, or a sudden shadow or curtain in the vision. Occasionally visual loss is gradual.

The client with glaucoma asks the nurse is complete vision will return. The most appropriate response is: A. "Although some vision as been lost and cannot be restored, further loss may be prevented by adhering to the treatment plan." B. "Your vision will return as soon as the medications begin to work." C. "Your vision will never return to normal." D. "Your vision loss is temporary and will return in about 3-4 weeks."

A Vision loss to glaucoma is irreparable. The client should be reassured that although some vision has been lost and cannot be restored, further loss may be prevented by adhering to the treatment plan. Option C does not provide reassurance to the client.

A clinic nurse is providing instructions to a client with glaucoma regarding the prescribed treatment measures for the disorder. The nurse prepares the instructions based on the primary objective of: a) maintaining intraocular pressure at a reduced level b) producing mydriasis c) increasing the formation of aqueous humor d) promoting dilation of the pupils of the eyes

A - The principle of treatment of the client with glaucoma is to maintain intraocular pressure at a reduced level to prevent further damage to intraocular structures. Medications are used to create miosis (constriction of the pupil) and reduce formation of the aqueous humor by the ciliary body.

Mike, a 43-year old construction worker, has a history of hypertension. He smokes two packs of cigarettes a day, is nervous about the possibility of being unemployed, and has difficulty coping with stress. His current concern is calf pain during minimal exercise that decreased with rest. The nurse assesses Mike's symptoms as being associated with peripheral arterial occlusive disease. The nursing diagnosis is probably: A Alteration in tissue perfusion related to compromised circulation B Dysfunctional use of extremities related to muscle spasms C Impaired mobility related to stress associated with pain D Impairment in muscle use associated with pain on exertion

A Alteration in tissue perfusion related to compromised circulation

Cancer can cause changes in what component of Virchow's triad? A Blood coagulability B Vessel walls C Blood flow D Blood viscosity

A Blood coagulability

A nurse is assessing the neurovascular status of a client who returned to the surgical nursing unit 4 hours ago after undergoing aortoiliac bypass graft. The affected leg is warm, and the nurse notes redness and edema. The pedal pulse is palpable and unchanged from admission. The nurse interprets that the neurovascular status is: A Normal because of the increased blood flow through the leg B Slightly deteriorating and should be monitored for another hour C Moderately impaired, and the surgeon should be called. D Adequate from the arterial approach, but venous complications are arising.

A Normal because of the increased blood flow through the leg Question 12 Explanation: An expected outcome of surgery is warmth, redness, and edema in the surgical extremity because of increased blood flow. Options 2, 3, and 4 are incorrect interpretations

A nurse has an order to begin administering warfarin sodium (coumadin) to a client. While implementing this order, the nurse ensures that which of the following medications is available on the nursing unit as the antidote for Coumadin? A Vitamin K B Aminocaproic acid C Potassium chloride D Protamine sulfate

A Vitamin K

Which person should the nurse identify as having the highest risk for abdominal aortic aneurysm? A 70-year-old male, with high cholesterol and hypertension A 40-year-old female with obesity and metabolic syndrome A 60-year-old male with renal insufficiency who is physically inactive A 65-year-old female with hyperhomocysteinemia and substance abuse

A 70-year-old male, with high cholesterol and hypertension The most common etiology of descending abdominal aortic aneurysm (AAA) is atherosclerosis. Male gender, age 65 years or older, and tobacco use are the major risk factors for AAAs of atherosclerotic origin. Other risk factors include the presence of coronary or peripheral artery disease, high blood pressure, and high cholesterol.

Which client is at greater risk for respiratory depression while receiving opioids for analgesia?

A child with an arm fracture and cystic fibrosis

Which client(s) would be appropriate to assign to a newly graduated RN, who has recently completed orientation? Choose all that apply.

A client second day post-op who needs pain medication prior to dressing changes

Which client(s) are appropriate to assign to the LPN/LVN, who will function under the supervision of the RN or team leader? (Choose all that apply.)

A client with a leg cast who needs neurologic checks and PRN hydrocodone A client post-op toe amputation with diabetic neuropathic pain

Which client is most likely to receive opioids for extended periods of time?

A client with progressive pancreatic cancer

Which are risk factors that are known to contribute to atherosclerosis-related diseases? (Select all that apply.) A. Low-density lipoprotein cholesterol (LDL-C) of 160 mg/dL B. Smoking C. Aspirin (acetylsalicylic acid [ASA]) consumption D. Type 2 diabetes E. Vegetarian diet

A, B, D Having an LDL-C value of less than 100 mg/dL is optimal; 100 to 129 mg/dL is near or less than optimal; with LDL-C 130 to 159 mg/dL (borderline high), the client is advised to modify diet and exercise. Smoking is a modifiable risk factor and should be avoided or terminated, and diabetes is a risk factor for atherosclerotic disease.

The nurse in the cardiology clinic is reviewing teaching about hypertension, provided at the client's last appointment. Which actions by the client indicate that teaching has been effective? (Select all that apply.) A. Has maintained a low-sodium, no-added-salt diet B. Has lost 3 pounds since last seen in the clinic C. Cooks food in palm oil to save money D. Exercises once weekly E. Has cut down on caffeine

A, B, E Clients with hypertension should consume low-sodium foods and should avoid adding salt. Weight loss can result in lower blood pressure. Caffeine promotes vasoconstriction, thereby elevating blood pressure.

A patient presenting to the ER with a hypertensive crisis (BP greater than 180/120), may have damage to which of the following? A. Brain B. Kidney C. Liver D. Heart E. Stomach F. Eyes

A, B. D. F CVA retinopathy heart failure renal failure IV beta blocker will be ordered immediately for a pt in a hypertensive crisis

The patient understand that which of the following are factors that he can change to decrease his risk of HTN? Select All That Apply A. smoking B. family history C. Alcohol consumption D. increased LDL E. Sedentary lifestyle

A, C. D, E Pt can change all but his family history

The silent killer, essential HTN, sometimes doesn't have obvious s/s, but some that may be reported by the patient include: Select All That Apply A. Dizziness B. Kidney disease C. Headache D. Syncope (fainting) E. Hot/flushed F. Nose bleed (epistaxis) G. Diabetes

A, C. D, E, F Kidney disease and diabetes are not s/s and are a factor in secondary HTN, not primary

For a client having an episode of acute narrow-angle glaucoma, a nurse expects to give which of the following medications? A Acetazolamide (Diamox) B Atropine C Furosemide (Lasix) D Urokinase (Abbokinase)

A. Acetazolamide, a carbonic anhydrase inhibitor, decreases intraocular pressure (IOP) by decreasing the secretion of aqueous humor. Atropine dilates the pupil and decreases outflow of aqueous humor, causing further increase in IOP. Lasix is a loop diuretic, and Urokinase is a thrombolytic agent; they aren't used for the treatment of glaucoma.

The nurse has notes that the physician has a diagnosis of presbycusis on the client's chart. The nurse plans care knowing the condition is: A sensorineural hearing loss that occurs with aging B A conductive hearing loss that occurs with aging. C Tinnitus that occurs with aging D Nystagmus that occurs with aging

A. Presbycusis is a type of hearing loss that occurs with aging. Presbycusis is a gradual sensorineural loss caused by nerve degeneration in the inner ear or auditory nerve.

For a client complaining of periocular aching after a surgical repair of a detached retina, which medication would be the most appropriate analgesic? A Acetaminophen B Codeine C Meperidine D Morphine

A. Because the discomfort is typically mild after surgery to repair a detached retina, a mild analgesic such as acetaminophen would be used. Codeine is constipating and may lead to straining and increased intraocular pressure (IOP). Meperidine often causes nausea and vomiting, further adding to the client's level of discomfort, and vomiting may lead to increased IOP. Morphine causes nausea, vomiting, and constipation, which should be avoided after surgery.

Mang Isko, a 68-year-old widower, has been stricken with cataracts about year ago. Which assessment date would the nurse expect when collecting the nursing history from the client? A Blurred vision B Eye pain C Floaters D Eye redness

A. Cataracts lead to progressive worsening and blurring of vision. Eye pain and redness, common with glaucoma, are not present with cataracts. Floaters are characteristics of retinal detachment.

The nurse is caring for a client with a diagnosis of detached retina. Which assessment sign would indicate that bleeding has occurred as a result of the retinal detachment? A Complaints of a burst of black spots or floaters B A sudden sharp pain in the eye C Total loss of vision D A reddened conjunctiva

A. Complaints of a sudden burst of black spots or floaters indicate that bleeding has occurred as a result of the detachment.

During the early postoperative period, the client who had a cataract extraction complains of nausea and severe eye pain over the operative site. The initial nursing action is to: A Call the physician B Administer the ordered main medication and antiemetic C Reassure the client that this is normal. D Turn the client on his or her operative side

A. Severe pain or pain accompanied by nausea is an indicator of increased intraocular pressure and should be reported to the physician immediately. The other options are inappropriate.

Which of the following symptoms would occur in a client with a detached retina? A Flashing lights and floaters B Homonymous hemianopia C Loss of central vision D Ptosis

A. Signs and symptoms of retinal detachment include abrupt flashing lights, floaters, loss of peripheral vision, or a sudden shadow or curtain in the vision. Occasionally visual loss is gradual.

The client with glaucoma asks the nurse is complete vision will return. The most appropriate response is: A "Although some vision as been lost and cannot be restored, further loss may be prevented by adhering to the treatment plan." B "Your vision will return as soon as the medications begin to work." C "Your vision will never return to normal." D "Your vision loss is temporary and will return in about 3-4 weeks."

A. Vision loss to glaucoma is irreparable. The client should be reassured that although some vision has been lost and cannot be restored, further loss may be prevented by adhering to the treatment plan. Option C does not provide reassurance to the client.

Mike, a 43-year old construction worker, has a history of hypertension. He smokes two packs of cigarettes a day, is nervous about the possibility of being unemployed, and has difficulty coping with stress. His current concern is calf pain during minimal exercise that decreased with rest. The nurse assesses Mike's symptoms as being associated with peripheral arterial occlusive disease. The nursing diagnosis is probably: a. Alteration in tissue perfusion related to compromised circulation b. Dysfunctional use of extremities related to muscle spasms c. Impaired mobility related to stress associated with pain d. Impairment in muscle use associated with pain on exertion.

A. Alteration in tissue perfusion related to compromised circulation

2. A patient has a 6-cm thoracic aortic aneurysm that was discovered during a routine chest x-ray. When obtaining an admission history from the patient, it will be most important for the nurse to ask about a. low back pain. b. trouble swallowing. c. abdominal tenderness. d. changes in bowel habits.

ANS: B Difficulty swallowing may occur with a thoracic aneurysm because of pressure on the esophagus. The other symptoms will be important to assess for in patients with abdominal aortic aneurysms.

28. A 46-year-old is diagnosed with thromboangiitis obliterans (Buerger's disease). When the nurse is developing a discharge teaching plan for the patient, which outcome has the highest priority for this patient? a. Cessation of all tobacco use b. Control of serum lipid levels c. Maintenance of appropriate weight d. Demonstration of meticulous foot care

ANS: A Absolute cessation of nicotine use is needed to reduce the risk for amputation in patients with Buerger's disease. Other therapies have limited success in treatment of this disease

22. An 80-year-old patient with a history of an abdominal aortic aneurysm arrives at the emergency department (ED) with severe back pain and absent pedal pulses. Which actions should the nurse take first? a. Obtain the blood pressure. b. Obtain blood for laboratory testing. c. Assess for the presence of an abdominal bruit. d. Determine any family history of kidney disease.

ANS: A Because the patient appears to be experiencing aortic dissection, the nurse's first action should be to determine the hemodynamic status by assessing blood pressure. The other actions also may be done, but they will not provide information that will determine what interventions are needed immediately for this patient.

4. A patient in the outpatient clinic has a new diagnosis of peripheral artery disease (PAD). Which group of medications will the nurse plan to include when providing patient teaching about PAD management? a. Statins b. Antibiotics c. Thrombolytics d. Anticoagulants

ANS: A Current research indicates that statin use by patients with PAD improves multiple outcomes. There is no research that supports the use of the other medication categories in PAD.

24. The nurse is caring for a patient immediately after repair of an abdominal aortic aneurysm. On assessment, the patient has absent popliteal, posterior tibial, and dorsalis pedis pulses. The legs are cool and mottled. Which action should the nurse take first? a. Notify the surgeon and anesthesiologist. b. Wrap both the legs in a warming blanket. c. Document the findings and recheck in 15 minutes. d. Compare findings to the preoperative assessment of the pulses.

ANS: A Lower extremity pulses may be absent for a short time after surgery because of vasospasm and hypothermia. Decreased or absent pulses together with a cool and mottled extremity may indicate embolization or graft occlusion. These findings should be reported to the physician immediately because this is an emergency situation. Because pulses are marked prior to surgery, the nurse would know whether pulses were present prior to surgery before notifying the health care providers about the absent pulses. Because the patient's symptoms may indicate graft occlusion or multiple emboli and a possible need to return to surgery, it is not appropriate to wait 15 minutes before taking action. A warming blanket will not improve the circulation to the patient's legs.

9. After teaching a patient with newly diagnosed Raynaud's phenomenon about how to manage the condition, which action by the patient demonstrates that the teaching has been effective? a. The patient exercises indoors during the winter months. b. The patient places the hands in hot water when they turn pale. c. The patient takes pseudoephedrine (Sudafed) for cold symptoms. d. The patient avoids taking nonsteroidal antiinflammatory drugs (NSAIDs).

ANS: A Patients should avoid temperature extremes by exercising indoors when it is cold. To avoid burn injuries, the patient should use warm, rather than hot, water to warm the hands. Pseudoephedrine is a vasoconstrictor, and should be avoided. There is no reason to avoid taking NSAIDs with Raynaud's phenomenon.

27. A patient who is 2 days post-femoral-popliteal bypass graft to the right leg is being cared for on the vascular unit. Which action by a licensed practical/vocational nurse (LPN/LVN) caring for the patient requires the registered nurse (RN) to intervene? a. The LPN/LVN has the patient sit in a chair for 90 minutes. b. The LPN/LVN assists the patient to walk 40 feet in the hallway. c. The LPN/LVN gives the ordered aspirin 160 mg after breakfast. d. The LPN/LVN places the patient in a Fowler's position for meals.

ANS: A The patient should avoid sitting for long periods because of the increased stress on the suture line caused by leg edema and because of the risk for venous thromboembolism (VTE). The other actions by the LPN/LVN are appropriate.

17. Which nursing action should be included in the plan of care after endovascular repair of an abdominal aortic aneurysm? a. Record hourly chest tube drainage. b. Monitor fluid intake and urine output. c. Check the abdominal incision for any redness. d. Teach the reason for a prolonged recovery period.

ANS: B Because renal artery occlusion can occur after endovascular repair, the nurse should monitor parameters of renal function such as intake and output. Chest tubes will not be needed for endovascular surgery, the recovery period will be short, and there will not be an abdominal wound.

8. When evaluating the discharge teaching for a patient with chronic peripheral artery disease (PAD), the nurse determines a need for further instruction when the patient says, "I will a. have to buy some loose clothes that do not bind across my legs or waist." b. use a heating pad on my feet at night to increase the circulation and warmth in my feet." c. change my position every hour and avoid long periods of sitting with my legs crossed." d. walk to the point of pain, rest, and walk again until the pain returns for at least 30 minutes 3 times a week."

ANS: B Because the patient has impaired circulation and sensation to the feet, the use of a heating pad could lead to burns. The other patient statements are correct and indicate that teaching has been successful.

26. The nurse is caring for a patient with critical limb ischemia who has just arrived on the nursing unit after having percutaneous transluminal balloon angioplasty. Which action should the nurse perform first? a. Begin oral intake. b. Obtain vital signs. c. Assess pedal pulses. d. Start discharge teaching.

ANS: B Bleeding is a possible complication after catheterization of the femoral artery, so the nurse's first action should be to assess for changes in vital signs that might indicate hemorrhage. The other actions are also appropriate but can be done after determining that bleeding is not occurring.

. When caring for a patient on the first postoperative day after an abdominal aortic aneurysm repair, which assessment finding is most important for the nurse to communicate to the health care provider? a. Presence of flatus b. Loose, bloody stools c. Hypoactive bowel sounds d. Abdominal pain with palpation

ANS: B Loose, bloody stools at this time may indicate intestinal ischemia or infarction, and should be reported immediately because the patient may need an emergency bowel resection. The other findings are normal on the first postoperative day after abdominal surgery.

7. The nurse performing an assessment with a patient who has chronic peripheral artery disease (PAD) of the legs and an ulcer on the right second toe would expect to find a. dilated superficial veins. b. swollen, dry, scaly ankles. c. prolonged capillary refill in all the toes. d. a serosanguineous drainage from the ulcer.

ANS: C Capillary refill is prolonged in PAD because of the slower and decreased blood flow to the periphery. The other listed clinical manifestations are consistent with chronic venous disease.

19. A 23-year-old patient tells the health care provider about experiencing cold, numb fingers when running during the winter and Raynaud's phenomenon is suspected. The nurse will anticipate teaching the patient about tests for a. hyperglycemia. b. hyperlipidemia. c. autoimmune disorders. d. coronary artery disease.

ANS: C Secondary Raynaud's phenomenon may occur in conjunction with autoimmune diseases such as rheumatoid arthritis. Patients should be screened for autoimmune disorders. Raynaud's phenomenon is not associated with hyperlipidemia, hyperglycemia, or coronary artery disease.

31. The nurse is caring for a patient with a descending aortic dissection. Which assessment finding is most important to report to the health care provider? a. Weak pedal pulses b. Absent bowel sounds c. Blood pressure 137/88 mm Hg d. 25 mL urine output over last hour

ANS: C The blood pressure is typically kept at less than 120 mm Hg systolic to minimize extension of the dissection. The nurse will need to notify the health care provider so that β-blockers or other antihypertensive medications can be prescribed. The other findings are typical with aortic dissection and should also be reported but do not require immediate action.

After receiving report, which patient admitted to the emergency department should the nurse assess first? a. 67-year-old who has a gangrenous left foot ulcer with a weak pedal pulse b. 58-year-old who is taking anticoagulants for atrial fibrillation and has black stools c. 50-year-old who is complaining of sudden "sharp" and "worst ever" upper back pain d. 39-year-old who has right calf tenderness, redness, and swelling after a long plane ride

ANS: C The patient's presentation is consistent with dissecting thoracic aneurysm, which will require rapid intervention. The other patients do not need urgent interventions.

30. Which nursing intervention for a patient who had an open repair of an abdominal aortic aneurysm 2 days previously is appropriate for the nurse to delegate to unlicensed assistive personnel (UAP)? a. Monitor the quality and presence of the pedal pulses. b. Teach the patient the signs of possible wound infection. c. Check the lower extremities for strength and movement. d. Help the patient to use a pillow to splint while coughing.

ANS: D Assisting a patient who has already been taught how to cough is part of routine postoperative care and within the education and scope of practice for UAP. Patient teaching and assessment of essential postoperative functions such as circulation and movement should be done by RNs.

35. The nurse reviews the admission orders shown in the accompanying figure for a patient newly diagnosed with peripheral artery disease. Which admission order should the nurse question? a. Use of treadmill for exercise b. Referral for dietary instruction c. Exercising to the point of discomfort d. Combined clopidogrel and omeprazole therapy

ANS: D Because the antiplatelet effect of clopidogrel is reduced when it is used with omeprazole, the nurse should clarify this prescription with the health care provider. The other interventions are appropriate for a patient with peripheral artery disease.

20. While working in the outpatient clinic, the nurse notes that a patient has a history of intermittent claudication. Which statement by the patient would support this information? a. "When I stand too long, my feet start to swell." b. "I get short of breath when I climb a lot of stairs." c. "My fingers hurt when I go outside in cold weather." d. "My legs cramp whenever I walk more than a block."

ANS: D Cramping that is precipitated by a consistent level of exercise is descriptive of intermittent claudication. Finger pain associated with cold weather is typical of Raynaud's phenomenon. Shortness of breath that occurs with exercise is not typical of intermittent claudication, which is reproducible. Swelling associated with prolonged standing is typical of venous disease.

3. Several hours after an open surgical repair of an abdominal aortic aneurysm, the UAP reports to the nurse that urinary output for the past 2 hours has been 40 mL. The nurse notifies the health care provider and anticipates an order for a(n) a. hemoglobin count. b. additional antibiotic. c. decrease in IV infusion rate. d. blood urea nitrogen (BUN) level.

ANS: D The decreased urine output suggests decreased renal perfusion, and monitoring of renal function is needed. There is no indication that infection is a concern, so antibiotic therapy and a WBC count are not needed. The IV rate may be increased because hypovolemia may be contributing to the patient's decreased urinary output.

6. A patient at the clinic says, "I have always taken a walk after dinner, but lately my leg cramps and hurts after just a few minutes of starting. The pain goes away after I stop walking, though." The nurse should a. check for the presence of tortuous veins bilaterally on the legs. b. ask about any skin color changes that occur in response to cold. c. assess for unilateral swelling, redness, and tenderness of either leg. d. assess for the presence of the dorsalis pedis and posterior tibial pulses.

ANS: D The nurse should assess for other clinical manifestations of peripheral arterial disease in a patient who describes intermittent claudication. Changes in skin color that occur in response to cold are consistent with Raynaud's phenomenon. Tortuous veins on the legs suggest venous insufficiency. Unilateral leg swelling, redness, and tenderness indicate venous thromboembolism (VTE).

5. A 73-year-old patient with chronic atrial fibrillation develops sudden severe pain, pulselessness, pallor, and coolness in the right leg. The nurse should notify the health care provider and immediately a. apply a compression stocking to the leg. b. elevate the leg above the level of the heart. c. assist the patient in gently exercising the leg. d. keep the patient in bed in the supine position.

ANS: D The patient's history and clinical manifestations are consistent with acute arterial occlusion, and resting the leg will decrease the oxygen demand of the tissues and minimize ischemic damage until circulation can be restored. Elevating the leg or applying an elastic wrap will further compromise blood flow to the leg. Exercise will increase oxygen demand for the tissues of the leg.

A 39-year-old woman with a history of smoking and oral contraceptive use is admitted with a venous thromboembolism (VTE) and prescribed unfractionated heparin. What laboratory test should the nurse review to evaluate the expected effect of the heparin? Platelet count Activated clotting time (ACT) International normalized ratio (INR) Activated partial thromboplastin time (APTT)

Activated partial thromboplastin time (APTT)

When titrating an analgesic to manage pain, what is the priority goal?

Administer smallest dose that provides relief with the fewest side effects.

The public health nurse is preparing to teach the members of the local swim club about care of the ears. Which of the following statements would be appropriate for the nurse to include? Use a clean cotton swab to dry the ear canal after swimming. A tight-fitting swim cap is preferred to earplugs for keeping water out of the ear. If there is debris in the ear canal, irrigate the ear with cold water. After an episode of acute inflammation, the swimmer should wait 7-10 days before returning to the water.

After an episode of acute inflammation, the swimmer should wait 7-10 days before returning to the water. Rationale: Guidelines include: Stay out of the water until the acute inflammatory process is completely resolved-ideally, 7-10 days before resuming water activities; and use silicone earplugs, and dry the outer ear with a towel, then use a hair dryer on the lowest setting several inches from the ear to dry the canal. Do not insert cotton swabs or other objects into the ear canal to dry it. A tight-fitting swim cap does not keep water out of the ear. Repeated exposure to cold water encourages the growth of exostoses in the ear canal.

A significant cause of venous thrombosis is: a. Altered blood coagulation b. Stasis of blood c. Vessel wall injury d. All of the above

All of the above

Mike, a 43-year old construction worker, has a history of hypertension. He smokes two packs of cigarettes a day, is nervous about the possibility of being unemployed, and has difficulty coping with stress. His current concern is calf pain during minimal exercise that decreased with rest. The nurse assesses Mike's symptoms as being associated with peripheral arterial occlusive disease. The nursing diagnosis is probably: a. Alteration in tissue perfusion related to compromised circulation b. Dysfunctional use of extremities related to muscle spasms c. Impaired mobility related to stress associated with pain d. Impairment in muscle use associated with pain on exertion.

Alteration in tissue perfusion related to compromised circulation

A client with diabetic neuropathy reports a burning, electrical-type in the lower extremities that is not responding to NSAIDs. You anticipate that the physician will order which adjuvant medication for this type of pain?

Amitriptyline (Elavil)

A first day postoperative client on a PCA pump reports that the pain control is inadequate. What is the first action you should take?

Assess the pain for location, quality, and intensity.

In caring for clients with pain and discomfort, which task is most appropriate to delegate to the nursing assistant?

Assist the client with preparation of a sitz bath.

A female patient with critical limb ischemia has had peripheral artery bypass surgery to improve her circulation. What care should the nurse provide on postoperative day 1? Keep the patient on bed rest. Assist the patient with walking several times. Have the patient sit in the chair several times. Place the patient on her side with knees flexed

Assist the patient with walking several times To avoid blockage of the graft or stent, the patient should walk several times on postoperative day 1 and subsequent days. Having the patient's knees flexed for sitting in a chair or in bed increase the risk of venous thrombosis and may place stress on the suture lines.

16.ID: 809567309 A female patient with critical limb ischemia has had peripheral artery bypass surgery to improve her circulation. What care should the nurse provide on postoperative day 1? Keep the patient on bed rest. Assist the patient with walking several times. Have the patient sit in the chair several times. Place the patient on her side with knees flexed.

Assist the patient with walking several times. avoid blockage of the graft or stent, the patient should walk several times on postoperative day 1 and subsequent days. Having the patient's knees flexed for sitting in a chair or in bed increase the risk of venous thrombosis and may place stress on the suture lines

When the patient who has undergone cataract extraction is preparing to go home, the nurse reinforces which of the following instructions? Remove the dressing at bedtime. A headache for the first few days after surgery should be expected. Avoid sleeping on the operative side. Set a new appointment with the surgeon for 1 month from today.

Avoid sleeping on the operative side. Rationale: The nurse should reinforce any limitation, such as avoiding reading, lifting, and strenuous activity, and sleeping on the operative side. The patient also needs to understand the importance of not disturbing the eye dressing, of follow-up appointments, and of manifestations of complications such as eye pain, change in vision, headache, or nausea.

When administering furosemide (Lasix) to a client who does not like bananas or orange juice, the nurse recommends that the client try which intervention to maintain potassium levels? A. Increase red meat in the diet. B. Consume melons and baked potatoes. C. Add several portions of dairy products each day. D. Try replacing your usual breakfast with oatmeal or Cream of Wheat.

B Melons and baked potatoes contain potassium. Red meat is high in saturated fat and is to be consumed sparingly. Dairy products are high in calcium. Cereals are fortified with iron; oatmeal contains fiber but not potassium

During the nursing history, which assessment data would the nurse expect the client scheduled for surgical correction of chronic open-angle glaucoma to report? A. Seeing flashes of lights and floaters B. Recent motor vehicle crash while changing lanes C. Complaints of headaches, nausea, and redness of the eyes D. Increasingly frequent episodes of double vision

B Typically, the client with chronic open-angle glaucoma experiences a gradual loss in peripheral vision leading to tunnel vision. Being involved in a motor vehicle crash while changing lanes suggests the disorder. The client may experience insidious blurring, decreased accommodation, mild aching eyes and, eventually, halos around the lights as intraocular pressure increases. Flashes of light and floaters are characteristic of retinal detachment. Nausea, headache, and eye redness are seen with an episode of acute (sudden) closed-angle closure. Double vision occurs when one eye has a lens and other is aphakic.

A nurse is assessing a client with an abdominal aortic aneurysm. Which of the following assessment findings by the nurse is probably unrelated to the aneurysm? A Pulsatile abdominal mass B Hyperactive bowel sounds in that area C Systolic bruit over the area of the mass D Subjective sensation of "heart beating" in the abdomen

B Hyperactive bowel sounds in that area

The client arrives in the emergency room after sustaining a chemical eye injury from a splash of battery acid. The initial nursing action is to: A. Begin visual acuity testing B. Irrigate the eye with sterile normal saline C. Swab the eye with antibiotic ointment D. Cover the eye with a pressure patch

B Emergency care following a chemical burn to the eye includes irrigating the eye immediately with sterile normal saline or ocular irrigating solution. In the emergency department, the irrigation should be maintained for at least 10 minutes. Following this emergency treatment, visual acuity is assessed.

The part of the ear that contains the receptors for hearing is the: A. Utricle B. Cochlea C. Middle ear D. Tympanic cavity

B The dendrites of the cochlear nerve terminate on the hair cells of the organ of Corti in the cochlea.

The nurse is performing a voice test to assess hearing. Which of the following describes the accurate procedure for performing this test? A. Stand 4 feet away from the client to ensure that the client can hear at this distance. B. Whisper a statement and ask the client to repeat it. C. Whisper a statement with the examiners back facing the client. D. Whisper a statement while the client blocks both ears

B The examiner stands 1-2 feet away from the client and asks the client to block one external ear canal. The nurse whispers a statement and asks the client to repeat it. Each ear is tested separately.

When using a Snellen alphabet chart, the nurse records the client's vision as 20/40. Which of the following statements best describes 20/40 vision? A. The client has alterations in near vision and is legally blind. B. The client can see at 20 feet what the person with normal vision can see at 40 feet. C. The client can see at 40 feet what the person with normal vision sees at 20 feet. D. The client has a 20% decrease in acuity in one eye, and a 40% decrease in the other eye

B The numerator refers to the client's vision while comparing the normal vision in the denominator.

Tonometry is performed on the client with a suspected diagnosis of glaucoma. The nurse analyzes the test results as documented in the client's chart and understands that normal intraocular pressure is: A. 2-7 mmHg B. 10-21 mmHg C. 22-30 mmHg D. 31-35 mmH

B Tonometry is the method of measuring intraocular fluid pressure using a calibrated instrument that indents or flattens the corneal apex. Pressures between 10 and 21 mmHg are considered within normal range.

The clinic nurse is preparing to test the visual acuity of a client using a Snellen chart. Which of the following identifies the accurate procedure for this visual acuity test? A. Both eyes are assessed together, followed by the assessment of the right and then the left eye. B. The right eye is tested followed by the left eye, and then both eyes are tested. C. The client is asked to stand at a distance of 40ft. from the chart and is asked to read the largest line on the chart. D. The client is asked to stand at a distance of 40ft from the chart and to read the line than can be read 200 ft away by an individual with unimpaired vision.

B Visual acuity is assessed in one eye at a time, and then in both eyes together with the client comfortably standing or sitting. The right eye is tested with the left eye covered; then the left eye is tested with the right eye covered. Both eyes then are tested together. Visual acuity is measured with or without corrective lenses and the client stands at a distance of 20ft. from the chart.

A nurse provides a list of instructions to a client with glaucoma regarding measures that will prevent an increase in intraocular pressure in the eyes. Which statement by the client indicates a need for further instructions? a) I can move objects weighing 20 pounds or more by pushing the object on the floor using my feet b) I can tie my shoelaces by bending over slowly c) I need to consume a diet high in fiber and bulk d) I need to maintain an intake of six to eight glasses of water a day

B - Activities such as bending over or straining at stool will increase intraocular pressure. The client needs to be instructed to maintain a diet high in bulk and fiber and to consume a high intake of liquids, unless contraindicated, to prevent constipation and straining at stools. The client should tie shoelaces by bending the knee, raising the thigh, and bringing the foot within reach. Objects weighing 20 pounds or more can be moved by pushing the object on the floor by using the feet or a mechanical dolly.

A client reports to the health care clinic for an eye examination, and a diagnosis of primary open-angle glaucoma is suspected. Which of the following nursing assessment questions will elicit information regarding the initial clinical manifestations associated with this disorder? a) do you have any pain in your eyes? b) have you had difficulty with peripheral vision? c) do bright lights causes a glare? d) is your central vision blurred?

B - Because glaucoma is usually symptom free, the client may first note changes in peripheral visual acuity. If pain occurs with glaucoma, it is usually late in the course of structural changes, with an intraocular pressure of 40 to 50 mm Hg or higher. Severe pain is characteristic of absolute glaucoma (total vision loss). Glare from bright lights is a complaint of a client with a cataract. Blurred central vision occurs with macular degeneration.

A nurse is caring for a client who had a percutaneous insertion of an inferior vena cava filter and was on heparin therapy before surgery. The nurse would inspect the surgical site most closely for signs of: A Thrombosis and infection B Bleeding and infection C Bleeding and wound dehiscence D Wound dehiscence and evisceration

B Bleeding and infection After inferior vena cava insertion, the nurse inspects the surgical site for bleeding and signs and symptoms of infection. Otherwise, care is the same as for any post-op client.

A client who has been receiving heparin therapy also is started on warfarin sodium (coumadin). The client asks the nurse why both medications are being administered. In formulating a response, the nurse incorporates the understanding that warfarin sodium: A Stimulates the breakdown of specific clotting factors by the liver, and it takes 2-3 days for this is exhibit an anticoagulant effect. B Inhibits synthesis of specific clotting factors in the liver, and it takes 3 to 4 days for this medication to exert an anticoagulation effect. C Stimulates production of the body's own thrombolytic substances, but it takes 2-4 days for it to begin. D Has the same mechanism action of heparin, and the crossover time is needed for the serum level of warfarin sodium to be therapeutic.

B Inhibits synthesis of specific clotting factors in the liver, and it takes 3 to 4 days for this medication to exert an anticoagulation effect. Warfarin sodium works in the liver and inhibits synthesis of four vitamin K-dependent clotting factors (X, IX, VII, and II), but it takes 3 to 4 days before the therapeutic effect of warfarin is exhibited.

In preparation for discharge of a client with arterial insufficiency and Raynaud's disease, client teaching instructions should include: A Walking several times each day as an exercise program. B Keeping the heat up so that the environment is warm C Wearing TED hose during the day D Using hydrotherapy for increasing oxygenation

B Keeping the heat up so that the environment is warm The client's instructions should include keeping the environment warm to prevent vasoconstriction. Wearing gloves, warm clothes, and socks will also be useful when preventing vasoconstriction, but TED hose would not be therapeutic. Walking would most likely increase pain

Buerger's disease is characterized by all of the following except: A Arterial thrombosis formation and occlusion B Lipid deposits in the arteries C Redness or cyanosis in the limb when it is dependent D Venous inflammation and occlusio

B Lipid deposits in the arteries

With peripheral arterial insufficiency, leg pain during rest can be reduced by: A Elevating the limb above heart level B Lowering the limb so it is dependent C Massaging the limb after application of cold compresses D Placing the limb in a plane horizontal to the body

B Lowering the limb so it is dependent

A client comes to the outpatient clinic and tells the nurse that he has had legs pains that began when he walks but cease when he stops walking. Which of the following conditions would the nurse assess for? A An acute obstruction in the vessels of the legs B Peripheral vascular problems in both legs C Diabetes D Calcium deficiency

B Peripheral vascular problems in both legs Question 16 Explanation: Intermittent claudication is a condition that indicates vascular deficiencies in the peripheral vascular system. If an obstruction were present, the leg pain would persist when the client stops walking. Low calcium levels may cause leg cramps but would not necessarily be related to walking.

Which technique is considered the gold standard for diagnosing DVT? A Ultrasound imaging B Venography C MRI D Doppler flow study

B Venography

A 24-year old man seeks medical attention for complaints of claudication in the arch of the foot. A nurse also notes superficial thrombophlebitis of the lower leg. The nurse would next assess the client for: A Familial tendency toward peripheral vascular disease B Smoking history C Recent exposures to allergens D History of insect bites

B Smoking history Question 18 Explanation: The mixture of arterial and venous manifestations (claudication and phlebitis, respectively) in the young male client suggests Buerger's disease. This is an uncommon disorder characterized by inflammation and thrombosis of smaller arteries and veins. This disorder typically is found in young adult males who smoke. The cause is not known precisely but is suspected to have an autoimmune component.

The nurse is performing a voice test to assess hearing. Which of the following describes the accurate procedure for performing this test? A Stand 4 feet away from the client to ensure that the client can hear at this distance. B Whisper a statement and ask the client to repeat it. C Whisper a statement with the examiners back facing the client. D Whisper a statement while the client blocks both ears.

B The examiner stands 1-2 feet away from the client and asks the client to block one external ear canal. The nurse whispers a statement and asks the client to repeat it. Each ear is tested separately.

Place the examples of drugs in the order of usage according to the World Health Organization (WHO) analgesic ladder. a. Morphine, hydromorphone, acetaminophen and lorazepam b. NSAIDs and corticosteroids c. Codeine, oxycodone and diphenhydramine

B, C, A

Which of the following control systems play a major role in maintaining blood pressure? Select All That Apply A. Renovascular system B. Arterial baroreceptor system C. Regulation of body fluid volume D. Respiratory System E. Renin-angiotensin-aldosterone system F. Vascular autoregulation G. Pulmonary system

B, C, E, F

Nerve deafness would most likely result from an injury or infection that damaged the: A Vagus nerve B Cochlear nerve C Vestibular nerve D Trigeminal nerve

B. Because the organ of hearing is the organ of Corti, located in the cochlea, nerve deafness would most likely accompany damage to the cochlear nerve.

A client arrives at the emergency room with a foreign body in the left ear that has been determined to be an insect. Which intervention would the nurse anticipate to be prescribed initially? A Irrigation of the ear B Instillation of diluted alcohol C Instillation of antibiotic ear drops D Instillation of corticosteroids ear drops

B. Insects are killed before removal unless they can be coaxed out by a flashlight or a humming noise. Mineral oil or diluted alcohol is instilled into the ear to suffocate the insect, which then is removed by using forceps. When the foreign object is vegetable matter, irrigation is not used because this material expands with hydration and the impaction becomes worse.

A client with Meniere's disease is experiencing severe vertigo. Which instruction would the nurse give to the client to assist in controlling the vertigo? A Increase fluid intake to 3000 ml a day B Avoid sudden head movements C Lie still and watch the television D Increase sodium in the diet

B. The nurse instructs the client to make slow head movements to prevent worsening of the vertigo. Dietary changes such as salt and fluid restrictions that reduce the amount of endolymphatic fluid sometimes are prescribed. Lying still and watching television will not control vertigo.

Otosclerosis is a common cause of conductive hearing loss. Which such a partial hearing loss: A Stapedectomy is the procedure of choice B Hearing aids usually restore some hearing C The client is usually unable to hear bass tones D Air conduction is more effective than bone conduction

B. With a partial hearing loss that auditory ossicles have not yet become fixed; as long as vibrations occur, a hearing aid may be beneficial.

A client who is complaining of tinnitus is describing a symptom that is: A Objective B Subjective C Functional D Prodromal

B. A subjective symptom such as ringing in the ears can be felt only by the client.

The part of the ear that contains the receptors for hearing is the: A Utricle B Cochlea C Middle ear D Tympanic cavity

B. The dendrites of the cochlear nerve terminate on the hair cells of the organ of Corti in the cochlea.

When using a Snellen alphabet chart, the nurse records the client's vision as 20/40. Which of the following statements best describes 20/40 vision? A The client has alterations in near vision and is legally blind. B The client can see at 20 feet what the person with normal vision can see at 40 feet. C The client can see at 40 feet what the person with normal vision sees at 20 feet. D The client has a 20% decrease in acuity in one eye, and a 40% decrease in the other eye.

B. The numerator refers to the client's vision while comparing the normal vision in the denominator.

During the nursing history, which assessment data would the nurse expect the client scheduled for surgical correction of chronic open-angle glaucoma to report? A Seeing flashes of lights and floaters B Recent motor vehicle crash while changing lanes C Complaints of headaches, nausea, and redness of the eyes D Increasingly frequent episodes of double vision

B. Typically, the client with chronic open-angle glaucoma experiences a gradual loss in peripheral vision leading to tunnel vision. Being involved in a motor vehicle crash while changing lanes suggests the disorder. The client may experience insidious blurring, decreased accommodation, mild aching eyes and, eventually, halos around the lights as intraocular pressure increases. Flashes of light and floaters are characteristic of retinal detachment. Nausea, headache, and eye redness are seen with an episode of acute (sudden) closed-angle closure. Double vision occurs when one eye has a lens and other is aphakic.

The clinic nurse is preparing to test the visual acuity of a client using a Snellen chart. Which of the following identifies the accurate procedure for this visual acuity test? A Both eyes are assessed together, followed by the assessment of the right and then the left eye. B The right eye is tested followed by the left eye, and then both eyes are tested. C The client is asked to stand at a distance of 40ft. from the chart and is asked to read the largest line on the chart. D The client is asked to stand at a distance of 40ft from the chart and to read the line than can be read 200 ft away by an individual with unimpaired vision.

B. Visual acuity is assessed in one eye at a time, and then in both eyes together with the client comfortably standing or sitting. The right eye is tested with the left eye covered; then the left eye is tested with the right eye covered. Both eyes then are tested together. Visual acuity is measured with or without corrective lenses and the client stands at a distance of 20ft. from the chart.

For a client diagnosed with epistaxis, which intervention would be included in the care plan? A Performing several abdominal thrust (Heimlich) maneuvers B Compressing the nares to the septum for 5 to 10 minutes C Applying an ice collar to the neck area D Encouraging warm saline throat gargles

B. When a client experiences epistaxis, the nurse should compress the soft outer portion of the nares against the septum for approximately 5 to 10 minutes. the client should sit upright, breathe through the mouth, and refrain from talking. Performing abdominal thrusts is appropriate for the client with a foreign-body aspiration. Applying an ice collar to the neck is commonly done for a client after a tonsillectomy. Warm saline throat gargles are appropriate for the client with pharyngitis.

With peripheral arterial insufficiency, leg pain during rest can be reduced by: a. Elevating the limb above heart level b. Lowering the limb so it is dependent c. Massaging the limb after application of cold compresses d. Placing the limb in a plane horizontal to the body

B. Lowering the limb so it is dependent

A 40-year-old man tells the nurse he has a diagnosis for the color and temperature changes of his limbs but can't remember the name of it. He says he must stop smoking and avoid trauma and exposure of his limbs to cold temperatures to get better. This description should allow the nurse to ask the patient if he has which diagnosis? Buerger's disease Venous thrombosis Acute arterial ischemia Raynaud's phenomenon

Beurgeer's disease is a nonatherosclerotic, segmental, recurrent inflammatory disorder of small and medium-sized veins and arteries of upper and lower extremities leading to color and temperature changes of the limbs, intermittent claudication, rest pain, and ischemic ulcerations. It primarily occurs in men younger than 45 years old with a long history of tobacco and/or marijuana use. Buerger's disease treatment includes smoking cessation, trauma and cold temperature avoidance, and a walking program. Venous thrombosis is the formation of a thrombus in association with inflammation of the vein. Acute arterial ischemia is a sudden interruption in arterial blood flow to a tissue caused by embolism, thrombosis, or trauma. Raynaud's phenomenon is characterized by vasospasm-induced color changes of the fingers, toes, ears, and nose.

For which client does the nurse recommend annual evaluation by an ophthalmologist? A. 35-year-old man with asthma B. 21-year-old man with psoriasis C. 24-year-old woman with diabetes D. 38-year-old woman who has lost 50 pounds

C

Which action by a nurse is most likely to increase accurate communication with a client who has low vision? A. Speaking slowly and loudly B. Enhancing the talk using hand gestures C. Being very specific with descriptions and directions D. Marking the door of the client's room to indicate his or her vision status

C

Which assessment is most important for the nurse to perform before instilling travoprost (Travatan) into the client's eye? A. Measuring the client's blood pressure B. Measuring the client's intraocular pressure C. Checking the cornea for abrasions or open areas D. Assessing heart rate and rhythm for 1 full minute

C

Essential hypertension would be diagnosed in a 40-year-old male whose blood pressure readings were consistently at or above which of the following? A. 120/ 90 mm Hg. B. 130/ 85 mm Hg. C. 140/ 90 mm Hg. D. 160/ 80 mm Hg.

C American Heart Association standards define hypertension as a consistent systolic blood pressure level greater than 140 mm Hg and a consistent diastolic blood pressure level greater than 90 mm Hg.

The nurse is developing a plan of care for the client scheduled for cataract surgery. The nurse documents which more appropriate nursing diagnosis in the plan of care? A. Self-care deficit B. Imbalanced nutrition C. Disturbed sensory perception D. Anxiety

C The most appropriate nursing diagnosis for the client scheduled for cataract surgery is Disturbed sensory perception (visual) related to lens extraction and replacement. Although the other options identify nursing diagnoses that may be appropriate, they are not related specifically to cataract surgery.

Which of the following does the nurse recognize as a contributing factor to high BP? A. decreased CO B. pulse rate of 100 C. increased afterload D. decreased stroke volume

C Increased afterload=increased PVR and BP = CO x PVR so if PVR increases then BP increases

The client undergoing femoral popliteal bypass states that he is fearful he will lose the limb in the near future. Which response by the nurse is most therapeutic? A. "Are you afraid you will not be able to work?" B. "If you control your diabetes, you can avoid amputation." C. "Your concerns are valid; we can review some steps to limit disease progression." D. "What about the situation concerns you most?"

C It is important to validate the client's concern and offer needed information. Asking the client if he is afraid may identify fear but does not allow the client to discuss his specific concern.

A client with peripheral arterial disease (PAD) has undergone percutaneous transluminal angioplasty (PTA) of the lower extremity. What is essential for the nurse to assess after the procedure? A. Ankle-brachial index B. Dye allergy C. Pedal pulses D. Gag reflex

C Priority nursing care focuses on assessment for bleeding at the arterial puncture site and monitoring for distal pulses. Pulse checks must be assessed postprocedure to detect improvement (stronger pulses) or complications (diminished or absent pulses).

The client sustains a contusion of the eyeball following a traumatic injury with a blunt object. Which intervention is initiated immediately? A. Notify the physician B. Irrigate the eye with cold water C. Apply ice to the affected eye D. Accompany the client to the emergency room

C Treatment for contusion begins at the time of injury. Ice is applied immediately. The client then should be seen by a physician and receive a thorough eye examination to rule out the presence of other eye injuries.

A nurse in the outpatient unit is preparing a client who is scheduled for a laser trabeculoplasty for the treatment of primary open-angle glaucoma. Which of the following instructions should the nurse provide to the client? a) the procedure takes about 2 hours b) activities can be resumed following the procedure immediately c) you may return to work 1 to 2 days following the procedure d) your vision loss will be restored following the procedure

C - Laser trabeculoplasty is performed in the outpatient setting and requires about 30 minutes. The client will experience little discomfort and may resume all normal activities including returning to work within 1 to 2 days. The treatment prevents further visual loss, but the lost vision cannot be restored.

When caring for a patient who has started anticoagulant therapy with warfarin (Coumadin), the nurse knows not to expect therapeutic benefits for: A At least 12 hours B The first 24 hours C 2-3 days D 1 week

C 2-3 days

The client is being discharged from the ambulatory care unit following cataract removal. The nurse provides instructions regarding home care. Which of the following, if stated by the client, indicates an understanding of the instructions? A. "I will take Aspirin if I have any discomfort." B. "I will sleep on the side that I was operated on." C. "I will wear my eye shield at night and my glasses during the day." D. "I will not lift anything if it weighs more that 10 pounds."

C The client is instructed to wear a metal or plastic shield to protect the eye from accidental and is instructed not to rub the eye. Glasses may be worn during the day. Aspirin or medications containing aspirin are not to be administered or taken by the client and the client is instructed to take acetaminophen as needed for pain. The client is instructed not to sleep on the side of the body on which the operation occurred. The client is not to lift more than 5 pounds.

The nurse is performing an admission assessment on a client with a diagnosis of detached retina. Which of the following is associated with this eye disorder? A. Pain in the affected eye B. Total loss of vision C. A sense of a curtain falling across the field of vision D. A yellow discoloration of the sclera

C A characteristic manifestation of retinal detachment described by the client is the feeling that a shadow or curtain is falling across the field of vision. No pain is associated with detachment of the retina. Options B and D are not characteristics of this disorder. A retinal detachment is an ophthalmic emergency and even more so if visual acuity is still normal.

The nurse is performing an assessment in a client with a suspected diagnosis of cataract. The chief clinical manifestation that the nurse would expect to note in the early stages of cataract formation is: A. Eye pain B. Floating spots C. Blurred vision D. Diplopia

C A gradual, painless blurring of central vision is the chief clinical manifestation of a cataract. Early symptoms include slightly blurred vision and a decrease in color perception.

In preparation for cataract surgery, the nurse is to administer prescribed eye drops. The nurse reviews the physicians orders, expecting which type of eye drops to be instilled? A. An osmotic diuretic B. A miotic agent C. A mydriatic medication D. A thiazide diuretic

C A mydriatic medication produces mydriasis or dilation of the pupil. Mydriatic medications are used preoperatively in the cataract client. These medication act by dilating the pupils. They also constrict blood vessels. An osmotic diuretic may be used to decrease intraocular pressure. A miotic medication constricts the pupil. A thiazide diuretic is not likely to be prescribed for a client with a cataract.

The client arrives in the emergency room with a penetrating eye injury from wood chips while cutting wood. The nurse assesses the eye and notes a piece of wood protruding from the eye, what is the initial nursing action? A. Remove the piece of wood using a sterile eye clamp B. Apply an eye patch C. Perform visual acuity tests D. Irrigate the eye with sterile saline

C If the laceration is the result of a penetrating injury, an object may be noted protruding from the eye. This object must never be removed except by the ophthalmologist because it may be holding ocular structures in place. Application of an eye patch or irrigation of the eye may disrupt the foreign body and cause further tearing of the sclera. (The only option that will prevent further disruption is to assess visual acuity.)

During a hearing assessment, the nurse notes that the sound lateralizes to the clients left ear with the Weber test. The nurse analyzes this result as: A. A normal finding B. A conductive hearing loss in the right ear C. A sensorineural or conductive loss D. The presence of nystagmus

C In the Weber tuning fork test the nurse places the vibrating tuning fork in the middle of the client's head, at the midline of the forehead, or above the upper lip over the teeth. Normally, the sound is heard in equally in both ears by bone conduction. If the client has a sensorineural hearing loss in one ear, the sound is heard in the other ear. If the client has a conductive hearing loss in one ear, the sound is heard in that ear.

Which of the following procedures or assessments must the nurse perform when preparing a client for eye surgery? A. Clipping the client's eyelashes B. Verifying the affected eye has been patched 24 hours before surgery C. Verifying the client has been NPO since midnight, or at least 8 hours before surgery. D. Obtaining informed consent with the client's signature and placing the forms on the chart.

C Maintaining NPO status for at least 8 hours before surgical procedures prevents vomiting and aspiration. There is no need to patch an eye before most surgeries or to clip the eyelashes unless specifically ordered by the physician. The physician is responsible for obtaining informed consent; the nurse validates that the consent is obtained.

Varicose veins can cause changes in what component of Virchow's triad? A Blood coagulability B Vessel walls C Blood flow D Blood viscosity

C Blood flow

Which of the following characteristics is typical of the pain associated with DVT? A Dull ache B No pain C Sudden onset D Tingling

C Sudden onset DVT is associated with deep leg pain of sudden onset, which occurs secondary to the occlusion. A dull ache is more commonly associated with varicose veins. A tingling sensation is associated with an alteration in arterial blood flow. If the thrombus is large enough, it will cause pain.

During a hearing assessment, the nurse notes that the sound lateralizes to the clients left ear with the Weber test. The nurse analyzes this result as: A A normal finding B A conductive hearing loss in the right ear C A sensorineural or conductive loss D The presence of nystagmus

C. In the Weber tuning fork test the nurse places the vibrating tuning fork in the middle of the client's head, at the midline of the forehead, or above the upper lip over the teeth. Normally, the sound is heard in equally in both ears by bone conduction. If the client has a sensorineural hearing loss in one ear, the sound is heard in the other ear. If the client has a conductive hearing loss in one ear, the sound is heard in that ear.

A 68-year-old client comes to the outpatient clinic and complains to the attending nurse his increased difficulty with "close-work" such as knitting. He indicates he does not have difficulty seeing objects on either side but does state that straight lines appear distorted or wavy. The nurse suspects which of the following disorders that is consistent with the client's reported symptoms? A Glaucoma B Cataracts C Macular degeneration D Subconjunctival hemorrhage

C. Macular degeneration, often age-related macular degeneration (AMD or ARMD), is a medical condition that usually affects older adults and results in a loss of vision in the center of the visual field (the macula) because of damage to the retina. It occurs in "dry" and "wet" forms. It is a major cause of blindness and visual impairment in older adults (>50 years). Macular degeneration can make it difficult or impossible to read or recognize faces, although enough peripheral vision remains to allow other activities of daily life.

A nurse is assessing a patient for essential hypertension. She will expect him to report which symptom? A. Chest tightness B. Shortness of Breath C. No symptoms to report D. Anxious

C. Primary (essential) HTN is the silent killer and s/s are not obvious

After attempting lifestyle changes with no improvement in the HTN, the nurse should expect the physician to prescribe which medication first? A. Calcium Channel Blocker B. ARB C. Thiazide diuretic D. Renin inhibitor

C. Thiazide diuretic is the first med to give, sometimes will be combined with a beta blocker. This combo is done so a lower dose of each med can be given.

The nurse is performing an admission assessment on a client with a diagnosis of detached retina. Which of the following is associated with this eye disorder? A Pain in the affected eye B Total loss of vision C A sense of a curtain falling across the field of vision D A yellow discoloration of the sclera.

C. A characteristic manifestation of retinal detachment described by the client is the feeling that a shadow or curtain is falling across the field of vision. No pain is associated with detachment of the retina. Options B and D are not characteristics of this disorder. A retinal detachment is an ophthalmic emergency and even more so if visual acuity is still normal.

The nurse is performing an assessment in a client with a suspected diagnosis of cataract. The chief clinical manifestation that the nurse would expect to note in the early stages of cataract formation is: A Eye pain B Floating spots C Blurred vision D Diplopia

C. A gradual, painless blurring of central vision is the chief clinical manifestation of a cataract. Early symptoms include slightly blurred vision and a decrease in color perception.

In preparation for cataract surgery, the nurse is to administer prescribed eye drops. The nurse reviews the physicians orders, expecting which type of eye drops to be instilled? A An osmotic diuretic B A miotic agent C A mydriatic medication D A thiazide diuretic

C. A mydriatic medication produces mydriasis or dilation of the pupil. Mydriatic medications are used preoperatively in the cataract client. These medication act by dilating the pupils. They also constrict blood vessels. An osmotic diuretic may be used to decrease intraocular pressure. A miotic medication constricts the pupil. A thiazide diuretic is not likely to be prescribed for a client with a cataract.

The nurse has conducted discharge teaching for a client who had a fenestration procedure for the treatment of otosclerosis. Which of the following, if stated by the client, would indicate that teaching was effective? A "I should drink liquids through a straw for the next 2-3 weeks." B "It's ok to take a shower and wash my hair." C "I will take stool softeners as prescribed by my doctor." D "I can resume my tennis lessons starting next week."

C. Following ear surgery, the client needs to avoid straining while having a bowel movement. The client needs to be instructed to avoid drinking through a straw for 2-3 weeks, air travel, and coughing excessively. The client needs to avoid getting his or her hair wet, washing hair, showering for 1 week, and rapidly moving the head, bouncing, and bending over for 3 weeks.

The ear bones that transmit vibrations to the oval window of the cochlea are found in the: A Inner ear B Outer ear C Middle ear D Eustachian tube

C. The bones in the middle ear transmit and amplify air pressure waves from the tympanic membrane to the oval window of the cochlea, which is the inner ear. The tympanic membrane separates the other from the middle ear.

Physiologically, the middle ear, containing the three ossicles, serves primarily to: A Maintain balance B Translate sound waves into nerve impulses C Amplify the energy of sound waves entering the ear D Communicate with the throat via the Eustachian tube.

C. The middle ear contains the three ossicles—malleus, incus, and stapes—which, along with the tympanic membrane and oval window, form an amplifying system.

A client is diagnosed with a disorder involving the inner ear. Which of the following is the most common client complaint associated with a disorder in this part of the ear? A Hearing loss B Pruritus C Tinnitus D Burning of the ear

C. Tinnitus is the most common complaint of clients with otological disorders, especially disorders involving the inner ear. Symptoms of tinnitus range from mild ringing in the ear, which can go unnoticed during the day, to a loud roaring in the ear, which can interfere with the client's thinking process and attention span.

The client sustains a contusion of the eyeball following a traumatic injury with a blunt object. Which intervention is initiated immediately? A Notify the physician B Irrigate the eye with cold water C Apply ice to the affected eye D Accompany the client to the emergency room

C. Treatment for contusion begins at the time of injury. Ice is applied immediately. The client then should be seen by a physician and receive a thorough eye examination to rule out the presence of other eye injuries.

The client is being discharged from the ambulatory care unit following cataract removal. The nurse provides instructions regarding home care. Which of the following, if stated by the client, indicates an understanding of the instructions? A "I will take Aspirin if I have any discomfort." B "I will sleep on the side that I was operated on." C "I will wear my eye shield at night and my glasses during the day." D "I will not lift anything if it weighs more that 10 pounds."

C. The client is instructed to wear a metal or plastic shield to protect the eye from accidental and is instructed not to rub the eye. Glasses may be worn during the day. Aspirin or medications containing aspirin are not to be administered or taken by the client and the client is instructed to take acetaminophen as needed for pain. The client is instructed not to sleep on the side of the body on which the operation occurred. The client is not to lift more than 5 pounds.

The nurse is developing a plan of care for the client scheduled for cataract surgery. The nurse documents which more appropriate nursing diagnosis in the plan of care? A Self-care deficit B Imbalanced nutrition C Disturbed sensory perception D Anxiety

C. The most appropriate nursing diagnosis for the client scheduled for cataract surgery is Disturbed sensory perception (visual) related to lens extraction and replacement. Although the other options identify nursing diagnoses that may be appropriate, they are not related specifically to cataract surgery.

The nurse is caring for a patient who has been receiving warfarin (Coumadin) and digoxin (Lanoxin) as treatment for atrial fibrillation. Because the warfarin has been discontinued before surgery, the nurse should diligently assess the patient for which complication early in the postoperative period until the medication is resumed? Decreased cardiac output Increased blood pressure Cerebral or pulmonary emboli Excessive bleeding from incision or IV sites

Cerebral or pulmonary emboli

The physician has ordered a placebo for a chronic pain client. You are newly hired nurse and you feel very uncomfortable administering the medication. What is the first action that you should take?

Contact the charge nurse for advice.

The nurse is reviewing the laboratory test results for a 68-year-old patient whose warfarin (Coumadin) therapy was terminated during the preoperative period. The nurse concludes that the patient is in the most stable condition for surgery after noting which INR (international normalized ratio) result? a. 1.0 b. 1.8 c. 2.7 d. 3.4

Correct Answer: A Rationale: The therapeutic range for INR is 2.0 to 3.0 for many clinical diagnoses. The larger the INR number, the greater the amount of anticoagulation. For this reason, the safest value before surgery is 1.0, meaning that the anticoagulation has been reversed.

A 45-year-old patient has breast cancer that has spread to the liver and spine. The patient has been taking oxycodone (OxyContin) and amitriptyline (Elavil) for pain control at home but now has constant severe pain and is hospitalized for pain control and development of a pain-management program. When doing the initial assessment, which question will be most appropriate to ask first? a. How would you describe your pain? b. How much medication do you take for the pain? c. How long have you had this pain? d. How many times a day do you medicate for pain?

Correct Answer: A Rationale: Because pain is a multidimensional experience, asking a question that addresses the patient's experience with the pain is likely to elicit more information than the more specific information asked in the other three responses. All of these questions are appropriate, but the response beginning "How would you describe your pain?" is the best first question. Cognitive Level: Application Text Reference: pp. 126, 131 Nursing Process: Assessment NCLEX: Physiological Integrity

A patient with chronic abdominal pain has learned to control the pain with the use of imagery and hypnosis. A family member asks the nurse how these techniques work. The nurse's reply will be based on the information that these strategies a. impact the cognitive and affective components of pain. b. prevent transmission of nociceptive stimuli to the cortex. c. increase the modulating effect of the efferent pathways. d. slow the release of transmitter chemicals in the dorsal horn.

Correct Answer: A Rationale: Cognitive therapies impact on the perception of pain by the brain rather than affecting efferent or afferent pathways or influencing the release of chemical transmitters in the dorsal horn. Cognitive Level: Comprehension Text Reference: p. 144 Nursing Process: Implementation NCLEX: Physiological Integrity

The health care provider tells a patient to use ibuprofen (Motrin, Advil) to relieve pain after treating a laceration on the patient's forearm from a dog bite. The patient asks the nurse how ibuprofen will control the pain. The nurse will teach the patient that ibuprofen interferes with the pain process by decreasing the a. production of pain-sensitizing chemicals. b. spinal cord transmission of pain impulses. c. sensitivity of the brain to painful stimuli. d. modulating effect of descending nerves.

Correct Answer: A Rationale: Nonsteroidal anti-inflammatory drugs (NSAIDs) provide analgesic effects by decreasing the production of pain-sensitizing chemicals such as prostaglandins at the site of injury. Transmission of impulses through the spinal cord, brain sensitivity to pain, and the descending nerve pathways are not affected by the NSAIDs. Cognitive Level: Application Text Reference: p. 128 Nursing Process: Implementation NCLEX: Physiological Integrity

A patient who uses a fentanyl (Duragesic) patch for chronic cancer pain complains to the nurse of the rapid onset of pain at a level 9 (of a 0-10 scale) and requests "something for pain that will work quickly." The best way for the nurse to document this information is as a. breakthrough pain. b. neuropathic pain. c. somatic pain. d. referred pain.

Correct Answer: A Rationale: Pain that occurs beyond the chronic pain already being treated by appropriate analgesics is termed breakthrough pain. Neuropathic pain is caused by damage to peripheral nerves or the central nervous system (CNS). Somatic pain is localized and arises from bone, joint, muscle, skin, or connective tissue. Referred pain is pain that is localized in uninjured tissue. Cognitive Level: Application Text Reference: p. 132 Nursing Process: Implementation NCLEX: Physiological Integrity

he hospice RN obtains the following information about a 72-year-old terminally ill patient with cancer of the colon. The patient takes oxycodone (OxyContin) 100 mg twice daily for level 6 abdomen pain on a 10-point scale. The pain has made it difficult to continue with favorite activities such as playing cards with friends twice a week. The patient's children are supportive of the patient's wish to stop chemotherapy but express sadness that the patient does not have long to live. Based on this information, which nursing diagnosis has priority in planning the patient's care? a. Impaired social interaction related to disabling pain b. Anxiety related to poor patient coping skills c. Disabled family coping related to patient-family conflict d. Risk for aspiration related to opioid use

Correct Answer: A Rationale: The assessment data indicate that the patient's priority is to be able to continue with favorite activities and that decreasing the pain level would accomplish this goal. There is no indication of anxiety, and the patient's and family's coping skills appear to be good. Although the patient is taking a large dose of oxycodone, there is no evidence that this has suppressed the respiratory rate or the gag/cough reflexes. Cognitive Level: Analysis Text Reference: p. 145

A hospice patient is in continuous pain, and the health care provider has left orders to administer morphine at a rate that controls the pain. When the nurse visits the patient, the patient is awake but moaning with severe pain and asks for an increase in the morphine dosage. The respiratory rate is 10 breaths per minute. The most appropriate action by the nurse is to a. titrate the morphine dose upward until the patient states there is adequate pain relief. b. administer a nonopioid analgesic, such as ibuprofen, to improve patient pain control. c. tell the patient that additional morphine can be administered when the respirations are 12. d. inform the patient that increasing the morphine will cause the respiratory drive to fail.

Correct Answer: A Rationale: The goal of opioid use in terminally ill patients is effective pain relief regardless of adverse effects such as respiratory depression. A nonopioid analgesic like ibuprofen would not provide adequate analgesia or be absorbed quickly. The rule of double effect provides ethical justification for administering an increased morphine dose to provide effective pain control even though the morphine may further decrease the patient's respiratory rate.

A patient is receiving morphine sulfate intravenously (IV) for right flank pain associated with a kidney stone in the right ureter. The patient also complains of right inner thigh pain and asks the nurse whether something is wrong with the right leg. In responding to the question, the nurse understands that the patient a. is experiencing referred pain from the kidney stone. b. has neuropathic pain from nerve damage caused by inflammation. c. has acute pain that may be progressing into chronic pain. d. is experiencing pain perception that has been affected by the morphine received earlier.

Correct Answer: A Rationale: The spread of pain to uninjured tissue is termed referred pain. Neuropathic pain refers to pain caused by nerve damage rather than by tissue injury or damage. When pain has lasted less than 3 months and is associated with an acute event (such as a kidney stone), it is acute pain. Morphine administration will decrease the perception of pain intensity, but it will not change the location of the pain. Cognitive Level: Application Text Reference: p. 129 Nursing Process: Assessment NCLEX: Physiological Integrity

A home health patient has a prescription for pentazocine (Talwin,) a mixed opioid agonist-antagonist. When teaching the patient and family about adverse effects, the nurse will plan to focus on how to monitor for a. agitation. b. respiratory depression. c. hypotension. d. physical dependence.

Correct Answer: A Rationale: This category of opioids causes more neurologic side effects, such as agitation, than the pure opioid opioids. The benefits to this category include less hypotension and respiratory depression and the absence of physical dependence. Cognitive Level: Application Text Reference: p. 138 Nursing Process: Planning NCLEX: Physiological Integrity

The health care provider plans to titrate a patient-controlled opioid infusion (PCA) to provide pain relief for a patient with acute surgical pain who has never received opioids in the past. Which of the following nursing actions regarding opioid administration are appropriate at this time? (Select all that apply.) a. Monitoring for therapeutic and adverse effects of opioid administration b. Teaching about the need to decrease opioid doses by the second postoperative day c. Assessing for signs that the patient is becoming addicted to the opioid d. Educating the patient about how analgesics improve postoperative activity level e. Emphasizing that the risk of opioid side effects increases over time

Correct Answer: A, D Rationale: Monitoring for pain relief and teaching the patient about how opioid use will improve postoperative outcomes are appropriate actions when administering opioids for acute pain. Although postoperative patients usually need decreasing amount of opioids by the second postoperative day, each patient's response is individual. Although tolerance may occur, addiction to opioids will not develop in the acute postoperative period. The patient should use the opioids to achieve adequate pain control, and so the nurse should not emphasize the adverse effects. Cognitive Level: Application Text Reference: pp. 138, 142 Nursing Process: Implementation NCLEX: Physiological Integrity

A 67-year-old man with peripheral artery disease is seen in the primary care clinic. Which symptom reported by the patient would indicate to the nurse that the patient is experiencing intermittent claudication? a Patient complains of chest pain with strenuous activity. b Patient says muscle leg pain occurs with continued exercise. c Patient has numbness and tingling of all his toes and both feet. d Patient states the feet become red if he puts them in a dependent position.

Correct Answer: B Intermittent claudication is an ischemic muscle ache or pain that is precipitated by a consistent level of exercise, resolves within 10 minutes or less with rest, and is reproducible. Angina is the term used to describe chest pain with exertion. Paresthesia is the term used to describe numbness or tingling in the toes or feet. Reactive hyperemia is the term used to describe redness of the foot; if the limb is in a dependent position the term is dependent rubor.

A patient with extensive second-degree burns on the legs and trunk is using patient-controlled analgesia (PCA) with IV morphine to be delivered at 1 mg every 10 minutes to control the pain. Several times during the night, the patient awakens in severe pain, and it takes more than an hour to regain pain relief. The most appropriate action by the nurse is to a. request that the health care provider order a bolus dose of morphine to be given when the patient awakens with pain. b. consult with the patient's health care provider about adding a continuous morphine infusion to the PCA regimen at night. c. teach the patient to push the button every 10 minutes for an hour before going to sleep even if the pain is minimal. d. administer a dose of morphine every 1 to 2 hours from the PCA machine while the patient is sleeping.

Correct Answer: B Rationale: Adding a continuous dose of the morphine at night will allow the patient to sleep without being awakened by the pain. Administering a dose of morphine when the patient awakens would not address the problem. Teaching the patient to administer unneeded medication before going to sleep might result in oversedation and respiratory depression. It is inappropriate for the nurse to administer the morphine while the patient sleeps because the nurse could not assess the pain level. Cognitive Level: Application Text Reference: p. 140 Nursing Process: Implementation NCLEX: Physiological Integrity

A patient being treated for chronic musculoskeletal pain tells the nurse, "I feel depressed because I can't even go out and play a round of golf." The patient describes the pain as "aching" and says it is usually at a level 7 of a scale of 1 to10. Based on these assessment data, which patient goal is most appropriate? After treatment, the patient will a. state that pain is at a level 2 of 10. b. be able to play 1 to 2 rounds of golf. c. exhibit fewer signs of depression. d. say that the aching has decreased.

Correct Answer: B Rationale: For chronic pain, patients are encouraged to set functional goals such as being able to perform daily activities and hobbies. The patient has identified playing golf as the desired activity, so a pain level of 2 or 10 or a decrease in aching would be less useful in evaluating successful treatment. The nurse should also assess for depression, but the patient has identified the depression as being due to the inability to play golf, so the goal of being able to play 1 or 2 rounds of golf is the most appropriate. Cognitive Level: Application Text Reference: p. 135 Nursing Process: Planning NCLEX: Physiological Integrity

A patient who has just started taking sustained-release morphine sulfate (MS Contin) for chronic pain complains of nausea and abdominal fullness. The most appropriate initial action by the nurse is to a. consult with the health care provider about using a different opioid. b. administer the ordered metoclopramide (Reglan) 10 mg IV. c. tell the patient that the nausea will subside in about a week. d. order the patient a clear liquid diet until the nausea decreases.

Correct Answer: B Rationale: Nausea is frequently experienced with the initiation of opioid therapy, and antiemetics are usually prescribed to treat this expected side effect. There is no indication that a different opioid is needed, although if the nausea persists, the health care provider may order a change of opioid. Although tolerance develops (in about a week), the nausea will subside; therefore, it would not be appropriate to allow the patient to continue to be nauseated. A clear liquid diet may decrease the nausea, but the best choice would be to administer the metoclopramide and allow the patient to eat. Cognitive Level: Application Text Reference: p. 138 Nursing Process: Implementation NCLEX: Physiological Integrity

A patient with chronic cancer pain experiences breakthrough pain (level 9 of 10) and anxiety while receiving sustained-release morphine sulfate (MS Contin) 160 mg every 12 hours. All these medications are ordered for the patient. Which one will be most appropriate for the nurse to administer first? a. Ibuprofen (Motrin) 400-800 mg orally b. Immediate-release morphine 30 mg orally c. Amitriptyline (Elavil) 10 mg orally. d. Lorazepam (Ativan) 1 mg orally

Correct Answer: B Rationale: The severe breakthrough pain indicates that the initial therapy should be a rapidly acting opioid, such as the immediate-release morphine. The ibuprofen and amitriptyline may be appropriate to use as adjuvant therapy, but they are not likely to block severe breakthrough pain. Use of anti-anxiety agents for pain control is inappropriate because this patient's anxiety for this patient is caused by the pain. Cognitive Level: Analysis Text Reference: pp. 137, 140 Nursing Process: Implementation NCLEX: Physiological Integrity

All the following medications are included in the admission orders for an 86-year-old patient with moderate degenerative arthritis in both hips. Which medication will the nurse use as an initial therapy? a. Aspirin (Bayer) 650 mg orally b. Oxycodone (Roxicodone) 5 mg orally c. Acetaminophen (Tylenol) 650 mg orally d. Naproxen (Aleve) 200 mg orally

Correct Answer: C Rationale: Acetaminophen is the best first-choice medication. The principle of "start low, go slow" is used to guide therapy when treating elderly adults because the ability to metabolize medications is decreased and the likelihood of medication interactions is increased. Non-opioid analgesics are used first, although opioids may be used later. Aspirin and the NSAIDs are associated with a high incidence of gastrointestinal bleeding in elderly patients. Cognitive Level: Application Text Reference: pp. 135, 148 Nursing Process: Implementation NCLEX: Physiological Integrity

The nurse is evaluating the effectiveness of imipramine (Tofranil), a tricyclic antidepressant, for a patient who is receiving the medication to help relieve chronic cancer pain. Which information is the best indicator that the imipramine is effective? a. The patient states, "I feel much less depressed since I've been taking the imipramine." b. The patient sleeps 8 hours every night. c. The patient says that the pain is manageable and that he or she can accomplish desired activities. d. The patient has no symptoms of anxiety.

Correct Answer: C Rationale: Imipramine is being used in this patient to manage chronic pain and improve functional ability. Although the medication is also prescribed for patients with depression, insomnia, and anxiety, the evaluation for this patient is based on improved pain control and activity level. Cognitive Level: Application Text Reference: p. 130

A patient receiving prn intermittent IV administration of opiates following gastric surgery watches a favorite television program every morning. The patient does not request pain medication during this time and when questioned denies the need for medication. The nurse's evaluation of this situation is that a. lying quietly in bed is the best method of controlling the patient's incisional pain. b. encouraging the patient to watch other television programs will decrease the pain. c. the distraction of the television enables the patient to decrease the perception of pain. d. the patient's dose of opiates needs to be decreased because her pain is well controlled.

Correct Answer: C Rationale: The distraction of watching a favorite program decreases the perception of pain by various brain structures. Immobilization may help to reduce pain, but it is not the best method for pain relief because immobility can lead to multiple postoperative complications. Other television programs are not likely to provide an adequate level of distraction. The patient will continue to require opioid analgesics when not watching the favorite program. Cognitive Level: Application Text Reference: p. 144 Nursing Process: Evaluation NCLEX: Physiological Integrity

When caring for a patient who is receiving epidural morphine, which information obtained by the nurse indicates that the patient may be experiencing a side effect of the medication? a. The patient complains of a "pounding" headache. b. The patient becomes restless and agitated. c. The patient has not voided for over 10 hours. d. The patient has cramping abdominal pain.

Correct Answer: C Rationale: Urinary retention is a common side effect of epidural opioids. Headache is not an anticipated side effect of morphine, although if there is a cerebrospinal fluid leak, the patient may develop a "spinal" headache. Sedation (rather than restlessness or agitation) would be a possible side effect. Hypotonic bowel sounds and constipation (rather than abdominal cramping) are concerns. Cognitive Level: Application Text Reference: p. 142 Nursing Process: Evaluation NCLEX: Physiological Integrity

A patient with a history of chronic cancer pain is admitted to the hospital. When reviewing the patient's home medications, which of these will be of most concern to the admitting nurse? a. Oxycodone (OxyContin) 80 mg twice daily b. Ibuprofen (Advil) 800 mg three times daily c. Amitriptyline (Elavil) 50 mg at bedtime d. Meperidine (Demerol) 25 mg every 4 hours

Correct Answer: D Rationale: Meperidine is contraindicated for chronic pain because it forms a metabolite that is neurotoxic and can cause seizures when used for prolonged periods. The ibuprofen, amitriptyline, and oxycodone are all appropriate medications for long-term pain management. Cognitive Level: Application Text Reference: p. 138 Nursing Process: Assessment NCLEX: Physiological Integrity

To obtain the most complete assessment data about a patient's chronic pain pattern, the nurse asks the patient a. "Can you describe where your pain is the worst?" b. "What is the intensity of your pain on a scale of 0 to 10?" c. "Would you describe your pain as aching, throbbing, or sharp?" d. "Can you describe your daily activities in relation to your pain?"

Correct Answer: D Rationale: The assessment of chronic pain should focus on the impact of the pain on patient function and daily activities. The other questions are also appropriate to ask, but will not give as complete information. Cognitive Level: Application Text Reference: p. 134 Nursing Process: Assessment NCLEX: Physiological Integrity

A postoperative patient who has undergone extensive bowel surgery moves as little as possible and does not use the incentive spirometer unless specifically reminded. The patient rates the pain severity as an 8 on a 10-point scale but tells the nurse, "I can tough it out." In encouraging the patient to use pain medication, the best explanation by the nurse is that a. very few patients become addicted to opioids when using them for acute pain control. b. there is little need to worry about side effects because these problems decrease over time. c. there are many pain medications and if one drug is ineffective, other drugs may be tried. d. unrelieved pain can be harmful due to the effect on respiratory function and activity level.

Correct Answer: D Rationale: The patient's low activity level, lack of spirometer use, and statement to the nurse indicate that there is a lack of understanding about the purpose of postoperative pain management. The patient did not indicate a concern about becoming addicted, a desire for alternate medications, or anxiety about analgesic side effects. Cognitive Level: Application Text Reference: pp. 126, 131, 145 Nursing Process: Implementation NCLEX: Physiological Integrity

The patient at highest risk for venous thromboembolism (VTE) is a. a 62-year-old man with spider veins who is having arthroscopic knee surgery. b. a 32-year-old woman who smokes, takes oral contraceptives, and is planning a trip to Europe. c. a 26-year-old woman who is 3 days postpartum and received maintenance IV fluids for 12 hours during her labor. d. an active 72-year-old man at home recovering from transurethral resection of the prostate for benign prostatic hyperplasia.

Correct answer: b Rationale: Three important factors (called Virchow's triad) in the etiology of venous thrombosis are (1) venous stasis, (2) damage of the endothelium (inner lining of the vein), and (3) hypercoagulability of the blood. Patients at risk for venous thrombosis usually have predisposing conditions for these three disorders (see Table 38-8). The 32-year-old woman has the highest risk: long trips without adequate exercise (venous stasis), tobacco use, and use of oral contraceptives. Note: The likelihood of hypercoagulability of blood is increased in women older than 35 years who use tobacco.

A patient with infective endocarditis develops sudden left leg pain with pallor, paresthesia, and a loss of peripheral pulses. The nurse's initial action should be to a. elevate the leg to promote venous return. b. start anticoagulant therapy with IV heparin. c. notify the physician of the change in peripheral perfusion. d. place the bed in reverse Trendelenburg to promote perfusion.

Correct answer: c Rationale: The patient has potentially developed acute arterial ischemia (sudden interruption in the arterial blood supply to the extremity), caused by an embolism from a cardiac thrombus that occurred as a complication of infective endocarditis. Clinical manifestations of acute arterial ischemia include any or all of the six Ps : pain, pallor, paralysis, pulselessness, paresthesia, and poikilothermia. Without immediate intervention, ischemia may progress quickly to tissue necrosis and gangrene within a few hours. If the nurse detects these signs, the physician should be notified immediately

The client who had cataract surgery with a lens implant 1 week ago remarks to the home care nurse that after his daughter left to go to her home in another state yesterday, he combined all of his prescribed eyedrops together in one container so he had fewer drops to administer. What is the nurse's best response? A. "This is not a good idea because not all of the drugs are on the same schedule." B. "That is a good idea; just remember to not touch the dropper to your eye when giving yourself the drops." C. "Call your surgeon immediately and get new prescriptions because together these drugs can lower your blood pressure." D. "Call your surgeon immediately and get new prescriptions to use one at a time because these drugs cannot be mixed together."

D

The nurse has conducted discharge teaching for a client who had a fenestration procedure for the treatment of otosclerosis. Which of the following, if stated by the client, would indicate that teaching was effective? A. "I can resume my tennis lessons starting next week." B. "I should drink liquids through a straw for the next 2-3 weeks." C. "It's ok to take a shower and wash my hair." D."I will take stool softeners as prescribed by my doctor."

D

The nurse is providing care to a client who has started warfarin after being diagnosed with a deep vein thrombosis. What health teaching will the nurse provide to the client related to self-management of warfarin therapy? A. "You must have your partial thromboplastin time checked every 2 weeks." B. "Massage the injection site after the warfarin is injected." C. "Eat plenty of dark green leafy vegetables while taking warfarin." D. "Report any signs of bleeding to your primary care provider."

D

With which client does the nurse avoid performing an otoscopic examination? A. 34-year-old woman who is pregnant B. 90-year-old woman who is visually impaired C. 75-year-old man with dizziness and vertigo D. 70-year-old man with advanced Alzheimer's disease

D

Which client who has just arrived in the emergency department does the nurse classify as emergent and needing immediate medical evaluation? A. A 60-year-old with venous insufficiency who has new-onset right calf pain and tenderness B. A 64-year-old with chronic venous ulcers who has a temperature of 100.1° F (37.8° C) C. A 69-year-old with a 40-pack-year cigarette history who is reporting foot numbness D. A 70-year-old with a history of diabetes who has "tearing" back pain and is diaphoretic

D The 70-year-old's history and clinical manifestations suggest possible aortic dissection. The nurse will immediately assess the client's blood pressure and plan for IV antihypertensive therapy, rapid diagnostic testing, and possible transfer to surgery

When developing a teaching session on glaucoma for the community, which of the following statements would the nurse stress? A. Glaucoma is easily corrected with eyeglasses B. White and Asian individuals are at the highest risk for glaucoma. C. Yearly screening for people ages 20-40 years is recommended. D. Glaucoma can be painless and vision may be lost before the person is aware of a problem

D Open-angle glaucoma causes a painless increase in intraocular pressure (IOP) with loss of peripheral vision. A variety of miotics and agents to decrease IOP and occasional surgery are used to treat glaucoma. Blacks have a threefold greater chance of developing with an increased chance of blindness than other groups. Individuals older than 40 should be screened.

Which of the following instruments is used to record intraocular pressure? A Goniometer B Ophthalmoscope C Slit lamp D Tonometer

D. A tonometer is a device used in glaucoma screening to record intraocular pressure. A goniometer measures joint movement and angles. An ophthalmoscope examines the interior of the eye, especially the retina. A slit lamp evaluates structures in the anterior chamber in the eye.

Which of the following instruments is used to record intraocular pressure? A. Goniometer B. Ophthalmoscope C. Slit lamp D. Tonometer

D A tonometer is a device used in glaucoma screening to record intraocular pressure. A goniometer measures joint movement and angles. An ophthalmoscope examines the interior of the eye, especially the retina. A slit lamp evaluates structures in the anterior chamber in the eye.

After the nurse instills atropine drops into both eyes for a client undergoing ophthalmic examination, which of the following instructions would be given to the client? A. "Be careful because the blink reflex is paralyzed." B. "Avoid wearing your regular glasses when driving." C. "Be aware that the pupils may be unusually small." D. "Wear dark glasses in bright light because the pupils are dilated.

D Atropine, an anticholinergic drug, has mydriatic effects causing pupil dilation. This allows more light onto the retina and may cause photophobia and blurred vision. Atropine doesn't paralyze the blink reflex or cause miosis (pupil constriction). Driving may be contraindicated to blurred vision.

Nurse Jairuz Roy is carrying out his preoperative teachings for an older client who will have cataract surgery on the right eye. The nurse concludes that the client needs further understanding about the teachings if he says: A. "I will sleep on my left side after the surgery." B. "I will wipe my nose gently if it is congested after surgery." C. "I will call my physician if I have sharp and sudden pain or a fever after surgery." D. "I will bend below my waist frequently to increase circulation after surgery."

D Immediately after the procedure, the client should avoid bending over, to prevent putting extra pressure on the eye.

The clinic nurse notes that the following several eye examinations, the physician has documented a diagnosis of legal blindness in the client's chart. The nurse reviews the results of the Snellen's chart test expecting to note which of the following? A. 20/20 vision B. 20/40 vision C. 20/60 vision D. 20/200 vision

D Legal blindness is defined as 20/200 or less with corrected vision (glasses or contact lenses) or visual acuity of less than 20 degrees of the visual field in the better eye.

The nurse is caring for a client that is hearing impaired. Which of the following approaches will facilitate communication? A. Speak frequently B. Speak loudly C. Speak directly into the impaired ear D. Speak in a normal tone

D Speaking in a normal tone to the client with impaired hearing and not shouting are important. The nurse should talk directly to the client while facing the client and speak clearly. If the client does not seem to understand what is said, the nurse should express it differently. Moving closer to the client and toward the better ear may facilitate communication, but the nurse should avoid talking directly into the impaired ear.

During eyedrop instillation, which intervention would the nurse perform to prevent systemic adverse effects from drug absorption? A. Applying pressure on the eyelid rim B. Having the client close his eyes tightly C. Placing the client in the supine position for a few minutes D. Applying pressure on the inner canthus

D Systemic absorption and subsequent adverse effects may occur if the medication enters the nasolacrimal canal. The nurse therefore applies pressure to the inner canthus, causing occlusion of this canal and minimizing the risk for systemic adverse effects. Applying pressure on the eyelid rim would not occlude this canal. Having the client close his eyes tightly may cause some of the medication to be expelled. Positioning has no effect on the blood flow of medication into the nasolacrimal canal and subsequent absorption.

The nurse is developing a teaching plan for the client with glaucoma. Which of the following instructions would the nurse include in the plan of care? A. Decrease fluid intake to control the intraocular pressure B. Avoid overuse of the eyes C. Decrease the amount of salt in the diet D. Eye medications will need to be administered lifelong

D The administration of eye drops is a critical component of the treatment plan for the client with glaucoma. The client needs to be instructed that medications will need to be taken for the rest of his or her life.

Aling Martha, a 73-year-old widow, tells to the nurse during the admission process that she was recently diagnosed with age-related hearing loss. Upon receiving such information, the nurse is correct if he suspects: A. Ménière's disease B. Otalgia C. Otitis media D. Presbycusis

D The term presbycusis refers to sensorineural hearing impairment in elderly individuals.

The client's vision is tested with a Snellen's chart. The results of the tests are documented as 20/60. The nurse interprets this as: A. The client can read at a distance of 60 feet what a client with normal vision can read at 20 feet. B. The client is legally blind. C. The client's vision is normal D. The client can read only at a distance of 20 feet what a client with normal vision can read at 60 feet.

D Vision that is 20/20 is normal, that is, the client is able to read from 20 feet what a person with normal vision can read from 20 feet. A client with a visual acuity of 20/60 only can read at a distance of 20 feet of what a person with normal vision can read at 60 feet.

Ben is diagnosed with a retinal detachment at the inner aspect of the right eye. Into which position would the nurse place the client? A. Fowler's position B. Supine with a small pillow C. Right-side lying D. Left-side lying

D When retinal detachment occurs, the client is positioned so that the area of detachment is dependent. For this client, the left-side lying position is used. Positioning the client in the Fowler, supine, or right-side lying position would not place the detached area in a dependent position.

male client has just had a cataract operation without a lens implant. In discharge teaching, the nurse will instruct the client's wife to: A. Feed him soft foods for several days to prevent facial movement B. Keep the eye dressing on for one week C. Have her husband remain in bed for 3 days D. Allow him to walk upstairs only with assistance

D Without a lens, the eye cannot accommodate. It is difficult to judge distance and climb stairs when the eyes cannot accommodate. Therefore, the client should walk up and down stairs only with assistance.

A significant cause of venous thrombosis is: A Altered blood coagulation B Stasis of blood C Vessel wall injury D All of the above

D All of the above

The most important factor in regulating the caliber of blood vessels, which determines resistance to flow, is: A Hormonal secretion B Independent arterial wall activity C The influence of circulating chemicals D The sympathetic nervous system

D The sympathetic nervous system

The nurse suspects that a client has developed an acute arterial occlusion of the right lower extremity based on which signs/symptoms? (Select all that apply.) A. Hypertension B. Tachycardia C. Bounding right pedal pulses D. Cold right foot E. Numbness and tingling of right foot F. Mottling of right foot and lower leg

D, E, F Pain, pallor, pulselessness, paresthesia, paralysis, poikilothermia (cool limb), and mottled color are characteristics of acute arterial occlusion.

Nurse Jairuz Roy is carrying out his preoperative teachings for an older client who will have cataract surgery on the right eye. The nurse concludes that the client needs further understanding about the teachings if he says: A "I will sleep on my left side after the surgery." B "I will wipe my nose gently if it is congested after surgery." C "I will call my physician if I have sharp and sudden pain or a fever after surgery." D "I will bend below my waist frequently to increase circulation after surgery."

D. Immediately after the procedure, the client should avoid bending over, to prevent putting extra pressure on the eye.

During eyedrop instillation, which intervention would the nurse perform to prevent systemic adverse effects from drug absorption? A Applying pressure on the eyelid rim B Having the client close his eyes tightly C Placing the client in the supine position for a few minutes D Applying pressure on the inner canthus

D. Systemic absorption and subsequent adverse effects may occur if the medication enters the nasolacrimal canal. The nurse therefore applies pressure to the inner canthus, causing occlusion of this canal and minimizing the risk for systemic adverse effects. Applying pressure on the eyelid rim would not occlude this canal. Having the client close his eyes tightly may cause some of the medication to be expelled. Positioning has no effect on the blood flow of medication into the nasolacrimal canal and subsequent absorption.

Aling Martha, a 73-year-old widow, tells to the nurse during the admission process that she was recently diagnosed with age-related hearing loss. Upon receiving such information, the nurse is correct if he suspects: A Ménière's disease B Otalgia C Otitis media D Presbycusis

D. The term presbycusis refers to sensorineural hearing impairment in elderly individuals.

Ben is diagnosed with a retinal detachment at the inner aspect of the right eye. Into which position would the nurse place the client? A Fowler's position B Supine with a small pillow C Right-side lying D Left-side lying

D. When retinal detachment occurs, the client is positioned so that the area of detachment is dependent. For this client, the left-side lying position is used. Positioning the client in the Fowler, supine, or right-side lying position would not place the detached area in a dependent position.

After the nurse instills atropine drops into both eyes for a client undergoing ophthalmic examination, which of the following instructions would be given to the client? A "Be careful because the blink reflex is paralyzed." B "Avoid wearing your regular glasses when driving." C "Be aware that the pupils may be unusually small." D "Wear dark glasses in bright light because the pupils are dilated."

D. Atropine, an anticholinergic drug, has mydriatic effects causing pupil dilation. This allows more light onto the retina and may cause photophobia and blurred vision. Atropine doesn't paralyze the blink reflex or cause miosis (pupil constriction). Driving may be contraindicated to blurred vision.

A nurse would question an order to irrigate the ear canal in which of the following circumstances? A Ear pain B Hearing loss C Otitis externa D Perforated tympanic membrane

D. Irrigation of the ear canal is contraindicated with perforation of the tympanic membrane because the solution entering the inner ear may cause dizziness, nausea, vomiting, and infection.

When developing a teaching session on glaucoma for the community, which of the following statements would the nurse stress? A Glaucoma is easily corrected with eyeglasses B White and Asian individuals are at the highest risk for glaucoma. C Yearly screening for people ages 20-40 years is recommended. D Glaucoma can be painless and vision may be lost before the person is aware of a problem.

D. Open-angle glaucoma causes a painless increase in intraocular pressure (IOP) with loss of peripheral vision. A variety of miotics and agents to decrease IOP and occasional surgery are used to treat glaucoma. Blacks have a threefold greater chance of developing with an increased chance of blindness than other groups. Individuals older than 40 should be screened.

Chad, a 5-year-old preschooler, is brought to the clinic due to an ear problem. Which assessment data would cause the nurse to suspect serous otitis media? A Bright red, bulging or retracted tympanic membrane and fever. B Inflammation of the external ear and crust formation on the auditory canal C Sensorineural hearing loss and complaints of tinnitus D Plugged feeling in the ear and reverberation of the client's own voice.

D. Serous otitis media is manifested by a plugged feeling in the ear, reverberation of the client's own voice, and hearing loss. A bright red, bulging or retracted tympanic membrane and fever suggest suppurative otitis media. Inflammation of the external ear and crust formation on the auditory canal suggest external otitis media. Sensorineural hearing loss and tinnitus indicate otosclerosis.

The nurse is caring for a client that is hearing impaired. Which of the following approaches will facilitate communication? A Speak frequently B Speak loudly C Speak directly into the impaired ear D Speak in a normal tone

D. Speaking in a normal tone to the client with impaired hearing and not shouting are important. The nurse should talk directly to the client while facing the client and speak clearly. If the client does not seem to understand what is said, the nurse should express it differently. Moving closer to the client and toward the better ear may facilitate communication, but the nurse should avoid talking directly into the impaired ear.

The nurse is developing a teaching plan for the client with glaucoma. Which of the following instructions would the nurse include in the plan of care? A Decrease fluid intake to control the intraocular pressure B Avoid overuse of the eyes C Decrease the amount of salt in the diet Eye medications will need to be administered lifelong.

D. The administration of eye drops is a critical component of the treatment plan for the client with glaucoma. The client needs to be instructed that medications will need to be taken for the rest of his or her life.

A male client has just had a cataract operation without a lens implant. In discharge teaching, the nurse will instruct the client's wife to: A Feed him soft foods for several days to prevent facial movement B Keep the eye dressing on for one week C Have her husband remain in bed for 3 days D Allow him to walk upstairs only with assistance

D. Without a lens, the eye cannot accommodate. It is difficult to judge distance and climb stairs when the eyes cannot accommodate. Therefore, the client should walk up and down stairs only with assistance.

The clinic nurse notes that the following several eye examinations, the physician has documented a diagnosis of legal blindness in the client's chart. The nurse reviews the results of the Snellen's chart test expecting to note which of the following? A 20/20 vision B 20/40 vision C 20/60 vision D 20/200 vision

D. Legal blindness is defined as 20/200 or less with corrected vision (glasses or contact lenses) or visual acuity of less than 20 degrees of the visual field in the better eye.

The client's vision is tested with a Snellen's chart. The results of the tests are documented as 20/60. The nurse interprets this as: A The client can read at a distance of 60 feet what a client with normal vision can read at 20 feet. B The client is legally blind. C The client's vision is normal D The client can read only at a distance of 20 feet what a client with normal vision can read at 60 feet.

D. Vision that is 20/20 is normal, that is, the client is able to read from 20 feet what a person with normal vision can read from 20 feet. A client with a visual acuity of 20/60 only can read at a distance of 20 feet of what a person with normal vision can read at 60 feet.

A significant cause of venous thrombosis is: a. Altered blood coagulation b. Stasis of blood c. Vessel wall injury d. All of the above

D. All

The nurse providing postoperative teaching for a patient who has undergone LASIK surgery advises the patient to contact the office if he experiences: Dry eyes. Undercorrection of visual acuity. Decreased night vision with halos. Drainage from the eye.

Drainage from the eye. Rationale: Following surgery, patients can experience a temporary loss of sharpness, over- or undercorrection of visual acuity, dry eyes, or temporarily decreased night vision with halos, glare, and starbursts. Bleeding or drainage could indicate a surgical complication, and should be reported immediately.

A preschool-age child is undergoing bilateral myringotomy with tubes for the treatment of recurrent otitis media. The mother asks the nurse what the tubes will do. The nurse's best response is that the tubes will: Provide for drainage. Prevent future episodes of otitis media. Equalize pressures between the middle ear and the environment. Prevent the spread of infection to the mastoid bone.

Equalize pressures between the middle ear and the environment. Rationale: Myringotomy (an incision of the tympanic membrane) is performed to relieve pressure caused by the edema and swelling of otitis media, which impairs the eustachian tubes' ability to equalize pressures between the middle ear and the environment. Without relief of pressure, the tympanic membrane can rupture, the delicate structures of the middle ear can be damaged, and infection can spread to adjoining tissues and structures. Infection can still occur and spread despite myringotomy.

A client with chronic pain reports to you, the charge nurse, that the nurse have not been responding to requests for pain medication. What is your initial action?

Have a conference with the nurses responsible for care of client

A male patient was admitted for a possible ruptured aortic aneurysm, but had no back pain. Ten minutes later his assessment includes the following: sinus tachycardia at 138, BP palpable at 65 mm Hg, increasing waist circumference, and no urine output. How should the nurse interpret this assessment about the patient's aneurysm? Tamponade will soon occur. The renal arteries are involved. Perfusion to the legs is impaired. He is bleeding into the abdomen.

He is bleeding into the abdomen.

19.ID: 809567305 A male patient was admitted for a possible ruptured aortic aneurysm, but had no back pain. Ten minutes later his assessment includes the following: sinus tachycardia at 138, BP palpable at 65 mm Hg, increasing waist circumference, and no urine output. How should the nurse interpret this assessment about the patient's aneurysm? Tamponade will soon occur. The renal arteries are involved. Perfusion to the legs is impaired. He is bleeding into the abdomen.

He is bleeding into the abdomen. The lack of back pain indicates the patient is most likely exsanguinating into the abdominal space, and the bleeding is likely to continue without surgical repair. A blockade of the blood flow will not occur in the abdominal space as it would in the retroperitoneal space where surrounding anatomic structures may control the bleeding. The lack of urine output does not indicate renal artery involvement, but that the bleeding is occurring above the renal arteries, which decreases the blood flow to the kidneys. There is no assessment data indicating decreased perfusion to the legs.

Which of the following suggestions should be taught by the nurse to the patient with tinnitus to assist the patient with coping with the symptom to promote concentration and sleep? Earplugs Sleeping medication Hearing aid that produces a tone to mask tinnitus Bedtime use of nasal decongestant spray

Hearing aid that produces a tone to mask tinnitus Rationale: Options for masking tinnitus to promote concentrate and sleep include ambient noise from a radio or sound system; masking device or white noise machine; hearing aid that produces a tone to mask the tinnitus; and hearing aid that amplifies ambient sound. Earplugs will focus the patient's attention on the sound. The patient's issue is the noise, not a sleeping disorder; hence, sleeping medications are not indicated. The tinnitus is a result of neurosensory damage, not of congestion.

. A 73-year-old man with dementia has a venous ulcer related to chronic venous insufficiency. The nurse should provide education on which type of diet for this patient and his caregiver? Low-fat diet High-protein diet Calorie-restricted diet High-carbohydrate diet

High-protein diet A patient with a venous ulcer should have a balanced diet with adequate protein, calories, and micronutrients; this type of diet is essential for healing. Nutrients most important for healing include protein, vitamins A and C, and zinc. Foods high in protein (e.g., meat, beans, cheese, tofu), vitamin A (green leafy vegetables), vitamin C (citrus fruits, tomatoes, cantaloupe), and zinc (meat, seafood) must be provided. Restricting fat or calories is not helpful for wound healing or in patients of normal weight. For overweight individuals with no active venous ulcer, a weight-loss diet should be considered.

A 32-year-old female is prescribed diltiazem (Cardizem) for Raynaud's phenomenon. To evaluate the patient's expected response to this medication, what is most important for the nurse to assess? Improved skin turgor Decreased cardiac rate Improved finger perfusion Decreased mean arterial pressure

Improved finger perfusion Raynaud's phenomenon is an episodic vasospastic disorder of small cutaneous arteries, most frequently involving the fingers and toes. Diltiazem (Cardizem) is a calcium channel blocker that relaxes smooth muscles of the arterioles by blocking the influx of calcium into the cells, thus reducing the frequency and severity of vasospastic attacks. Perfusion to the fingertips is improved and vasospastic attacks reduced. Diltiazem may decrease heart rate and blood pressure, but that is not the purpose in Raynaud's phenomenon. Skin turgor is most often a reflection of hydration status.

The patient with glaucoma has been prescribed travoprost (Travatan), a prostaglandin analog, to improve the outflow of aqueous humor and reduce ocular pressure. Of the following signs or symptoms, which will the nurse teach the patient to report immediately to the eye doctor? Burning in the eye when the eye drops are used Tearing in the eye when the eye drops are used Changes in the color of the eyes Intolerable pain in the eyes after the drops are used

Intolerable pain in the eyes after the drops are used Rationale: Prostaglandin analogs have some adverse effects, such as blurred vision and stinging, and, when used long-term, cause permanent darkening of the iris of the eye and eyebrows, increased growth of eyelashes, and conjunctival redness.

Which of the following physical assessment maneuvers will allow the nurse to differentiate the ear pain of a patient with otitis externa from the ear pain of a patient with otitis media? Manipulation of the auricle Irrigating the ear canal with cool water Rinne and Weber tests Whisper test

Manipulation of the auricle Rationale: The pain of otitis externa can be differentiated from that associated with otitis media by the manipulation of the auricle. In external otitis, this maneuver increases pain, whereas the patient with otitis media experiences no change in pain perception. Rinne and Weber tests are used to diagnose sensorineural hearing loss. The caloric test (irrigation with cool water) evaluates vestibulo-ocular reflex. The whisper test evaluates hearing loss.

Which of the following aspects for health teaching regarding acute and chronic conjunctivitis is the most important information to be emphasized to a group of preschool parents? Preventable cause of blindness Importance of early medical diagnosis Mode of transmission Pain management

Mode of transmission Rationale: The nurse should emphasize to the group ways to prevent the transmission of the infection. Conjunctivitis is highly contagious, and can be transmitted by direct contact with contaminated fomites (e.g., hands, tissues, towels, pillow cases), and some forms can be transmitted to an infant during vaginal delivery. Prevention is the most important aspect of conjunctivitis to be communicated to the group. Pain with acute conjunctivitis is not common.

1. When discussing risk factor modification for a 63-year-old patient who has a 5-cm abdominal aortic aneurysm, the nurse will focus discharge teaching on which patient risk factor? a. Male gender b. Turner syndrome c. Abdominal trauma history d. Uncontrolled hypertension

NS: D All of the factors contribute to the patient's risk, but only hypertension can potentially be modified to decrease the patient's risk for further expansion of the aneurysm.

For a cognitively impaired client who cannot accurately report pain, what is the first action that you should take?

Obtain baseline behavioral indicators from family members.

Which route of administration is preferable for administration of daily analgesics (if all body systems are functional)?

Oral

A 67-year-old man with peripheral artery disease is seen in the primary care clinic. Which symptom reported by the patient would indicate to the nurse that the patient is experiencing intermittent claudication? Patient complains of chest pain with strenuous activity. Patient says muscle leg pain occurs with continued exercise. Patient has numbness and tingling of all his toes and both feet. Patient states the feet become red if he puts them in a dependent position.

Patient says muscle leg pain occurs with continued exercise. Intermittent claudication is an ischemic muscle ache or pain that is precipitated by a consistent level of exercise, resolves within 10 minutes or less with rest, and is reproducible. Angina is the term used to describe chest pain with exertion. Paresthesia is the term used to describe numbness or tingling in the toes or feet. Reactive hyperemia is the term used to describe redness of the foot; if the limb is in a dependent position the term is dependent rubor.

The nurse is preparing to administer a scheduled dose of enoxaparin (Lovenox) 30 mg subcutaneously. What should the nurse do to administer this medication correctly? Remove the air bubble in the prefilled syringe. Aspirate before injection to prevent IV administration. Rub the injection site after administration to enhance absorption. Pinch the skin between the thumb and forefinger before inserting the needle.

Pinch the skin between the thumb and forefinger before inserting the needle.

The nurse's first action for a patient who presents to the Emergency Department with manifestations of corneal perforation is to: Contact the patient's ophthalmologist. Place the patient in a supine position. Cover the eye with gauze. Administer tears for dry eyes.

Place the patient in a supine position. Rationale: If corneal perforation is suspected, the nurse should first place the patient in the supine position, then close the eye, and cover it with a dry, sterile dressing. Thereafter, the nurse should notify either the patient's ophthalmologist or an on-call ophthalmologist to see the patient. The complications of corneal perforation include infection of deep eye structures and loss of ocular contents. Nothing should be placed in the eye.

What medications should the nurse expect to include in the teaching plan to decrease the risk of cardiovascular events and death for PAD patients (select all that apply)? Ramipril (Altace) Cilostazol (Pletal) Simvastatin (Zocor) Clopidogrel (Plavix) Warfarin (Coumadin) Aspirin (acetylsalicylic acid)

Ramipril (Altace) Simvastatin (Zocor Aspirin (acetylsalicylic acid) Angiotensin-converting enzyme inhibitors (e.g., ramipril [Altace]) are used to control hypertension. Statins (e.g., simvastatin [Zocor]) are used for lipid management. Aspirin is used as an antiplatelet agent. Cilostazol (Pletal) is used for intermittent claudication, but it does not reduce CVD morbidity and mortality risks. Clopidogrel may be used if the patient cannot tolerate aspirin. Anticoagulants (e.g., warfarin [Coumadin]) are not recommended to prevent CVD events in PAD patients.

13. Assessment of a patient's peripheral IV site reveals that phlebitis has developed over the past several hours. Which intervention should the nurse implement first? Remove the patient's IV catheter. Apply an ice pack to the affected area. Decrease the IV rate to 20 to 30 mL/hr. Administer prophylactic anticoagulants.

Remove the patient's IV catheter.

Which non-pharmacological measure is particularly useful for a client with acute pancreatitis

Side-lying position with knees to chest and pillow against abdomen

The patient is in the preoperative holding area, having been brought to surgery from the Emergency Department because of a detached retina. The tear has occurred in the right posterior aspect of the retina. The nurse places the patient in which of the following positions? At a 45-degree angle (semi-Fowler's position) At a 90-degree angle (high Fowler's position) Prone Supine

Supine Rationale: The patient's head is positioned so that gravity pulls the detached portion of the retina into closer contact with the choroid. In this example, the patient's head should be flat and turned slightly to the right.

A spry 77-year-old female patient has been diagnosed with simple glaucoma. She lives alone, takes medications for asthma, is on a fixed income, and walks to Catholic Mass on Sunday mornings. The nurse must be certain that the ophthalmologist is aware of which of the aforementioned facts about the patient? Lives alone. Takes medications for asthma. Lives on a fixed income. Walks to Catholic Mass on Sundays.

Takes medications for asthma. Rationale: Living alone and a fixed income should be concerns addressed through a Social Services consultation. The patient could be listed on a clergy visit list if hospitalized. Most importantly, the nurse wants to be certain that the patient takes medications for asthma because beta-adrenergic blockers are used topically to treat glaucoma. In asthmatic patients, beta-adrenergic blockers can cause bronchospasm, and are contraindicated

The most important factor in regulating the caliber of blood vessels, which determines resistance to flow, is: a. Hormonal secretion b. Independent arterial wall activity. c. The influence of circulating chemicals d. The sympathetic nervous system

The sympathetic nervous system

A 23 year old man is admitted to the ambulatory surgery unit to undergo removal of a cholesteatoma from the middle ear. Which of the following post surgery nursing interventions takes priority? Assessment of hearing Treatment of nausea Document decreased hearing acuity Position client in supine on the affected side

Treatment of nausea Rationale: Clients who undergo ear surgery should be reassured that decreased hearing acuity immediately after surgery is expected. The client should be placed in a side-lying position on the unaffected side with the head of the bed elevated. Antiemetics should be administered to prevent vomiting since vomiting may increase the pressure in the middle ear, disrupting the surgical site.

20.ID: 809567339 The patient had aortic aneurysm repair. What priority nursing action will the nurse use to maintain graft patency? Assess output for renal dysfunction. Use IV fluids to maintain adequate BP. Use oral antihypertensives to maintain cardiac output. Maintain a low BP to prevent pressure on surgical site

Use IV fluids to maintain adequate BP. The priority is to maintain an adequate BP (determined by the surgeon) to maintain graft patency. A prolonged low BP may result in graft thrombosis, and hypertension may cause undue stress on arterial anastomoses resulting in leakage of blood or rupture at the suture lines, which is when IV antihypertensives may be used. Renal output will be assessed when the aneurysm repair is above the renal arteries to assess graft patency, not maintain it.

The patient had aortic aneurysm repair. What priority nursing action will the nurse use to maintain graft patency? Assess output for renal dysfunction. Use IV fluids to maintain adequate BP. Use oral antihypertensives to maintain cardiac output. Maintain a low BP to prevent pressure on surgical site

Use IV fluids to maintain adequate BP. The priority is to maintain an adequate BP (determined by the surgeon) to maintain graft patency. A prolonged low BP may result in graft thrombosis, and hypertension may cause undue stress on arterial anastomoses resulting in leakage of blood or rupture at the suture lines, which is when IV antihypertensives may be used. Renal output will be assessed when the aneurysm repair is above the renal arteries to assess graft patency, not maintain it.

The nurse is caring for a patient who has a hearing deficit. Which of the following techniques should be implemented by the nurse to improve communication? (Select all that apply.) Keep the patient's face in full light. Use a low voice pitch with normal loudness. Speak at a slower rate than usual. Use short sentences.

Use a low voice pitch with normal loudness. Use short sentences. Rationale: Techniques to improve communication with a patient with a hearing loss include keeping one's face in full light-placing light in the patient's face will prevent patient from seeing the speaker; using a low voice pitch with normal loudness; speaking at a normal rate, and not overarticulating; and using short sentences, and pausing at the end of each sentence.

The nurse describes tinnitus to the family members of a client who was recently diagnosed with tinnitus, and indicates that the noise in the ears does not come from the environment. The nurse recommends which of the following strategies to the family to improve the client's ability to concentrate and sleep? Stand directly in front of the client diagnosed with tinnitus before speaking. Provide a quiet environment. Use a white-noise machine. Advise the client to investigate surgical treatment for the disease.

Use a white-noise machine Rationale: The nurse must never assume that a client who has developed tinnitus has a hearing loss, an assumption that unlies Answer 1. Since the sound does not come from the environment, ensuring a "quiet" atmosphere does not help the client. Surgery is not used in the treatment of tinnitus. "White noise" is a constant, nondisturbing noise that masks the tinnitus. Machines are made for the purpose of masking not only tinnitus, but snoring sounds by a partner while sleeping.

The nurse is admitting a 68-year-old preoperative patient with a suspected abdominal aortic aneurysm (AAA). The medication history reveals that the patient has been taking warfarin (Coumadin) on a daily basis. Based on this history and the patient's admission diagnosis, the nurse should prepare to administer which medication? Vitamin K Cobalamin Heparin sodium Protamine sulfate

Vitamin K Coumadin is a Vitamin K antagonist anticoagulant that could cause excessive bleeding during surgery if clotting times are not corrected before surgery. For this reason, vitamin K is given as the antidote for warfarin (Coumadin).

When the visiting nurse is teaching the family of a patient with acute conjunctivitis regarding the disease process, the nurse emphasizes which of the following statements as most important? Wash hands before and after instilling eye drops. Cleanse the patient's contact lenses twice each day. Ask patient to gently rub eyes to circulate the eye drops after instillation. Use a cotton-tipped swab to cleanse the right eye and then the left eye.

Wash hands before and after instilling eye drops. Rationale: Acute conjunctivitis is highly contagious. Teach the family to wash hands thoroughly before and after instilling eye medication. Contact lens use should be avoided until the infection has cleared. Rubbing the eyes increases the risk of corneal trauma. Using a new swab between eyes prevents cross-contamination.

8. The nurse is testing pupillary reaction to light. What should be observed in the direct and consensual pupils when a light is shoe obliquely into one eye? A. Direct pupil constricts, consensual pupil constricts B. Direct pupil constricts consensual pupil dilates? C. Direct pupil dilates, consensual pupil dilates D. Direct pupil dilates, consensual pupil constricts

a

A nurse is assessing the neurovascular status of a client who returned to the surgical nursing unit 4 hours ago after undergoing aortoiliac bypass graft. The affected leg is warm, and the nurse notes redness and edema. The pedal pulse is palpable and unchanged from admission. The nurse interprets that the neurovascular status is: a. Normal because of the increased blood flow through the leg b. Slightly deteriorating and should be monitored for another hour c. Moderately impaired, and the surgeon should be called. d. Adequate from the arterial approach, but venous complications are arising.

a. An expected outcome of surgery is warmth, redness, and edema in the surgical extremity because of increased blood flow. Options 2, 3, and 4 are incorrect interpretations.

Cancer can cause changes in what component of Virchow's triad? a. Blood coagulability b. Vessel walls c. Blood flow d. Blood viscosity

a. Blood coagulability

A nurse has an order to begin administering warfarin sodium (coumadin) to a client. While implementing this order, the nurse ensures that which of the following medications is available on the nursing unit as the antidote for Coumadin? a. Vitamin K b. Aminocaproic acid c. Potassium chloride d. Protamine sulfate

a. The antidote to warfarin (Coumadin) is Vitamin K and should be readily available for use if excessive bleeding or hemorrhage should occur.

1. Where are sound waves transmitted to after they pass through the eardrum? A. to the inner ear though the round window B. To the inner ear through the oval window? C. To the middle ear through the round window? D. To the middle ear through the Eustachian tube?

b

3. The nurse is performing a Snellen chart exam. The client missed two letters on the 20/30 line. Which of the following demonstrates appropriate documentation of the score? A. 20/30 B. 20/30-2 C. 20/40 D. 20/40-2

b

7. While performing an otoscopic exam, the nurse notices white spots on the client's tympanic membrane. What should the nurse do next? A. Report this finding to the physician? B. Document the finding in the client's chart? C. Assess the patient for Sensorineural hearing loss? D. Palpate the preauricular lymph nodes for tenderness?

b

21. When developing a teaching plan for a 76-year-old patient newly diagnosed with peripheral artery disease (PAD), which instructions should the nurse include? a. "Exercise only if you do not experience any pain." b. "It is very important that you stop smoking cigarettes." c. "Try to keep your legs elevated whenever you are sitting." d. "Put elastic compression stockings on early in the morning."

b. "It is very important that you stop smoking cigarettes." ANS: B Smoking cessation is essential for slowing the progression of PAD to critical limb ischemia and reducing the risk of myocardial infarction and death. Circulation to the legs will decrease if the legs are elevated. Patients with PAD are taught to exercise to the point of feeling pain, rest, and then resume walking. Support hose are not used for patients with PAD.

A nurse is caring for a client who had a percutaneous insertion of an inferior vena cava filter and was on heparin therapy before surgery. The nurse would inspect the surgical site most closely for signs of: a. Thrombosis and infection b. Bleeding and infection c. Bleeding and wound dehiscence. d. Wound dehiscence and evisceration.

b. After inferior vena cava insertion, the nurse inspects the surgical site for bleeding and signs and symptoms of infection. Otherwise, care is the same as for any post-op client.

In preparation for discharge of a client with arterial insufficiency and Raynaud's disease, client teaching instructions should include: a. Walking several times each day as an exercise program. b. Keeping the heat up so that the environment is warm c. Wearing TED hose during the day d. Using hydrotherapy for increasing oxygenation

b. The client's instructions should include keeping the environment warm to prevent vasoconstriction. Wearing gloves, warm clothes, and socks will also be useful when preventing vasoconstriction, but TED hose would not be therapeutic. Walking would most likely increase pain.

A 24-year old man seeks medical attention for complaints of claudication in the arch of the foot. A nurse also notes superficial thrombophlebitis of the lower leg. The nurse would next assess the client for: a. Familial tendency toward peripheral vascular disease b. Smoking history c. Recent exposures to allergens d. History of insect bites

b. The mixture of arterial and venous manifestations (claudication and phlebitis, respectively) in the young male client suggests Buerger's disease. This is an uncommon disorder characterized by inflammation and thrombosis of smaller arteries and veins. This disorder typically is found in young adult males who smoke. The cause is not known precisely but is suspected to have an autoimmune component.

A 24-year old man seeks medical attention for complaints of claudication in the arch of the foot. A nurse also notes superficial thrombophlebitis of the lower leg. The nurse would next assess the client for: a. Familial tendency toward peripheral vascular disease b. Smoking history c. Recent exposures to allergens d. History of insect bites

b. The mixture of arterial and venous manifestations (claudication and phlebitis, respectively) in the young male client suggests Buerger's disease. This is an uncommon disorder characterized by inflammation and thrombosis of smaller arteries and veins. This disorder typically is found in young adult males who smoke. The cause is not known precisely but is suspected to have an autoimmune component.

Which are probable clinical findings in a person with an acute lower extremity VTE (select all that apply)? a.Pallor and coolness of foot and calf b.Mild to moderate calf pain and tenderness c.Grossly diminished or absent pedal pulses d.Unilateral edema and induration of the thigh e.Palpable cord along a superficial varicose vein

b.Mild to moderate calf pain and tenderness d.Unilateral edema and induration of the thigh The patient with lower extremity venous thromboembolism (VTE) may or may not have unilateral leg edema, extremity pain, a sense of fullness in the thigh or calf, paresthesias, warm skin, erythema, or a systemic temperature greater than 100.4 F (38 C). If the calf is involved, it may be tender to palpation.

Which are probable clinical findings in a person with an acute lower extremity VTE (select all that apply)? a.Pallor and coolness of foot and calf b.Mild to moderate calf pain and tenderness c.Grossly diminished or absent pedal pulses d.Unilateral edema and induration of the thigh e.Palpable cord along a superficial varicose vein

b.Mild to moderate calf pain and tenderness d.Unilateral edema and induration of the thigh The patient with lower extremity venous thromboembolism (VTE) may or may not have unilateral leg edema, extremity pain, a sense of fullness in the thigh or calf, paresthesias, warm skin, erythema, or a systemic temperature greater than 100.4 F (38 C). If the calf is involved, it may be tender to palpation.

Which clinical manifestations are seen in patients with either Buerger's disease or Raynaud's phenomenon (select all that apply)? a.Intermittent fevers b.Sensitivity to cold temperatures c.Gangrenous ulcers on fingertips d.Color changes of fingers and toes e.Episodes of superficial vein thrombosis

b.Sensitivity to cold temperatures c.Gangrenous ulcers on fingertips d.Color changes of fingers and toes Both Buerger's disease and Raynaud's phenomenon have the following clinical manifestations in common: cold sensitivity, ischemic and gangrenous ulcers on fingertips, and color changes of the distal extremity (fingers or toes).

A patient is admitted to the hospital with a diagnosis of abdominal aortic aneurysm. Which signs and symptoms would suggest that his aneurysm has ruptured? a.Sudden shortness of breath and hemoptysis b.Sudden, severe low back pain and bruising along his flank c.Gradually increasing substernal chest pain and diaphoresis d.Sudden, patchy blue mottling on feet and toes and rest pain

b.Sudden, severe low back pain and bruising along his flank The clinical manifestations of a ruptured abdominal aortic aneurysm include severe back pain, back or flank ecchymosis (Grey Turner's sign), and hypovolemic shock (tachycardia, hypotension, pale clammy skin, decreased urine output, altered level of consciousness, and abdominal tenderness).

A patient is admitted to the hospital with a diagnosis of abdominal aortic aneurysm. Which signs and symptoms would suggest that his aneurysm has ruptured? a.Sudden shortness of breath and hemoptysis b.Sudden, severe low back pain and bruising along his flank c.Gradually increasing substernal chest pain and diaphoresis d.Sudden, patchy blue mottling on feet and toes and rest pain

b.Sudden, severe low back pain and bruising along his flank The clinical manifestations of a ruptured abdominal aortic aneurysm include severe back pain, back or flank ecchymosis (Grey Turner's sign), and hypovolemic shock (tachycardia, hypotension, pale clammy skin, decreased urine output, altered level of consciousness, and abdominal tenderness).

The patient at highest risk for venous thromboembolism (VTE) is a.a 62-year-old man with spider veins who is having arthroscopic knee surgery. b.a 32-year-old woman who smokes, takes oral contraceptives, and is planning a trip to Europe. c.a 26-year-old woman who is 3 days postpartum and received maintenance IV fluids for 12 hours during her labor. d.an active 72-year-old man at home recovering from transurethral resection of the prostate for benign prostatic hyperplasia.

b.a 32-year-old woman who smokes, takes oral contraceptives, and is planning a trip to Europe. Three important factors (called Virchow's triad) in the etiology of venous thrombosis are (1) venous stasis, (2) damage of the endothelium (inner lining of the vein), and (3) hypercoagulability of the blood. Patients at risk for venous thrombosis usually have predisposing conditions for these three disorders (see Table 38-8). The 32-year-old woman has the highest risk: long trips without adequate exercise (venous stasis), tobacco use, and use of oral contraceptives. Note: The likelihood of hypercoagulability of blood is increased in women older than 35 years who use tobacco.

The patient at highest risk for venous thromboembolism (VTE) is a.a 62-year-old man with spider veins who is having arthroscopic knee surgery. b.a 32-year-old woman who smokes, takes oral contraceptives, and is planning a trip to Europe. c.a 26-year-old woman who is 3 days postpartum and received maintenance IV fluids for 12 hours during her labor. d.an active 72-year-old man at home recovering from transurethral resection of the prostate for benign prostatic hyperplasia.

b.a 32-year-old woman who smokes, takes oral contraceptives, and is planning a trip to Europe. Three important factors (called Virchow's triad) in the etiology of venous thrombosis are (1) venous stasis, (2) damage of the endothelium (inner lining of the vein), and (3) hypercoagulability of the blood. Patients at risk for venous thrombosis usually have predisposing conditions for these three disorders (see Table 38-8). The 32-year-old woman has the highest risk: long trips without adequate exercise (venous stasis), tobacco use, and use of oral contraceptives. Note: The likelihood of hypercoagulability of blood is increased in women older than 35 years who use tobacco.

2. A client complains of noticing halos around the eyes. The nurse recognizes this as a symptom of which of these conditions? A. Retinal detachment B. Optic atrophy C. Narrow-angle glaucoma D. Ophthalmic migraines

c

3. The client complains of hearing loss on the right side of one-week duration. Which of these is the most likely explanation? A. Sensorineural loss? B. Subjective vertigo? C. Otitis externa? D. Postauricular cyst?

c

4. Which action will assist the nurse in bringing the optic disc into view? A. Decrease the beam of light? B. Move the ophthalmoscope towards the nose? C. Ask the client to look straight ahead? D. Use the examiners dominant eye?

c

5. While performing the cover test, the nurse notices an inward turn of the client's right eye. Which of these demonstrates proper documentation of this deviation? A. Exotropia of the right eye with cover test B. Presbyopia of the right eye with cover test C. Estropia of the right eye with cover test D. Strabismus of right eye with cover test

c

6. Which of these demonstrates proper use of the otoscope when examining the ears of an adult? A. Release the auricle once the speculum is inserted B. Angle the speculum towards the back of the ear canal C. Firmly but gently pull the auricle out, up and back. D. Use a small speculum to avoid pressure on the canal

c

9. The nurse is performing an ophthalmoscopic exam to detect changes in the retinal vessels. What condition does the nurse suspect when the vein appears to stop short on either side of the artery? A. Diabetes Mellitus B. Hyperthyroidism C. Hypertension D. Retinopathy

c

Varicose veins can cause changes in what component of Virchow's triad? a. Blood coagulability b. Vessel walls c. Blood flow d. Blood viscosity

c. Blood flow

Which of the following characteristics is typical of the pain associated with DVT? a. Dull ache b. No pain c. Sudden onset d. Tingling

c. DVT is associated with deep leg pain of sudden onset, which occurs secondary to the occlusion. A dull ache is more commonly associated with varicose veins. A tingling sensation is associated with an alteration in arterial blood flow. If the thrombus is large enough, it will cause pain.

A 50-year-old woman weighs 95 kg and has a history of tobacco use, high blood pressure, high sodium intake, and sedentary lifestyle. When developing an individualized care plan for her, the nurse determines that the most important risk factors for peripheral artery disease (PAD) that need to be modified are a.weight and diet. b.activity level and diet. c.tobacco use and high blood pressure. d.sedentary lifestyle and high blood pressure.

c. tobacco use and high blood pressure Significant risk factors for peripheral artery disease include tobacco use, hyperlipidemia, elevated levels of high-sensitivity C-reactive protein, diabetes mellitus, and uncontrolled hypertension; the most important is tobacco use. Other risk factors include family history, hypertriglyceridemia, hyperuricemia, increasing age, obesity, sedentary lifestyle, and stress.

A 50-year-old woman weighs 95 kg and has a history of tobacco use, high blood pressure, high sodium intake, and sedentary lifestyle. When developing an individualized care plan for her, the nurse determines that the most important risk factors for peripheral artery disease (PAD) that need to be modified are a.weight and diet. b.activity level and diet. c.tobacco use and high blood pressure. d.sedentary lifestyle and high blood pressure.

c. tobacco use and high blood pressure Significant risk factors for peripheral artery disease include tobacco use, hyperlipidemia, elevated levels of high-sensitivity C-reactive protein, diabetes mellitus, and uncontrolled hypertension; the most important is tobacco use. Other risk factors include family history, hypertriglyceridemia, hyperuricemia, increasing age, obesity, sedentary lifestyle, and stress.

8. The nurse has completed the Weber test on an elderly client and found lateralization of sound to the right ear. What is the client most likely experiencing? A. Perceptive hearing loss to the right ear? B. Sensorineural hearing loss to the left ear? C. External hearing loss to the left ear? D. Conductive hearing loss to the right ear?

d

Intravenous heparin therapy is ordered for a client. While implementing this order, a nurse ensures that which of the following medications is available on the nursing unit? a. Vitamin K b. Aminocaproic acid c. Potassium chloride d. Protamine sulfate

d. The antidote to heparin is protamine sulfate and should be readily available for use if excessive bleeding or hemorrhage should occur

A client is admitted with a venous stasis leg ulcer. A nurse assesses the ulcer, expecting to note that the ulcer: a. Has a pale colored base b. Is deep, with even edges c. Has little granulation tissue d. Has brown pigmentation around it.

d. Venous leg ulcers, also called stasis ulcers, tend to be more superficial than arterial ulcers, and the ulcer bed is pink. The edges of the ulcer are uneven, and granulation tissue is evident. The skin has a brown pigmentation from accumulation of metabolic waste products resulting from venous stasis. The client also exhibits peripheral edema. (options 1, 2, and 3 is due to tissue malnutrition; and thus us an arterial problem)

Which of the nursing interventions are appropriate to use during the postoperative care of the client who has undergone ear surgery? (Select all that apply.) instructing the client to keep the mouth open during sneezing. instructing the client to forcibly exhale with the mouth, nose, and glottis closed. administer antiemetic medication at the first sign of nausea. administer pain medication at the first sign of nausea. place client on the affected side. place client on the unaffected side.

instructing the client to keep the mouth open during sneezing administer antiemetic medication at the first sign of nausea place client on the unaffected side Rationale: Performing the Valsalva maneuver (forcibly exhaling with the mouth, nose, and glottis closed) will increase the pressure on the tympanic membrane which is undesirable. While pain can cause nausea in some cases, the most appropriate first-line medication for nausea in the postoperative client who has undergone ear surgery is an antiemetic. Vomiting increases pressure in the middle ear, disrupting the surgical site. The client is placed on the unaffected side in order to minimize the pressure in the middle ear.

From the following list of signs and symptoms, identify those which are associated with angle closure (also called narrow-angle or closed-angle) glaucoma. (Select all that apply.) painless, gradual loss of visual fields nausea and vomiting normal visual acuity seeing halos around lights

nausea and vomiting seeing halos around lights Rationale: Open angle glaucoma is painless, with gradual loss of visual fields. Visual acuity remains normal. Angle closure (also called narrow-angle or closed-angle) glaucoma is a medical emergency characterized by: nausea and vomiting, seeing colored halos around lights, a decrease in visual acuity, general malaise, and eye and face pain.

The client who has undergone eye surgery arrives at the postanesthesia care unit. The nurse observes sanguineous drainage on the eye dressing. The nurse's first action is to replace the dressing. notify the client's surgeon. lower the head of the bed. instill eye drops as ordered.

notify the client's surgeon. Rationale: Bleeding (sanguineous fluid) may indicate a surgical complication. The client's surgeon should be notified. Surgical dressings applied by the ophthalmologist will be removed by the ophthalmologist. The head of the bed should remain in semi to high Fowler's position to reduce intraocular pressure. Eye drops, even if ordered, should not be given until the bleeding is assessed.

A 62-year-old Hispanic male patient with diabetes mellitus has been diagnosed with peripheral artery disease (PAD). The patient is a smoker and has a history of gout. What should the nurse focus her teaching on to prevent complications for this patient? Gender Smoking Ethnicity Co-morbidities

smoking Smoking is the most significant factor for this patient. PAD is a marker of advanced systemic atherosclerosis. Therefore tobacco cessation is essential to reduce PAD progression, CVD events, and mortality. Diabetes mellitus and hyperuricemia are also risk factors. Being male or Hispanic are not risk factors for PAD.

The client is admitted to the acute care unit for surgery. A labyrinthectomy will be performed on the client's left ear. Which of the following post-procedure nursing interventions must take priority: standing on the client's unaffected side to speak to the client. assess for signs of infection. use a pad and pencil to communicate to the client. advise the client that the client's vertigo is gone.

standing on the client's unaffected side to speak to the client Rationale: The surgery of last resort for Méniére's disease is a labyrinthectomy. When removed, cochlear function is destroyed causing the client to be permanently deaf in the affected ear. The procedure relieves vertigo in nearly all cases. Infection is not apparent in the immediate postoperative period. There is no indication that the client has bilateral deafness.

accommodation

the process by which the eye's lens changes shape to focus near or far objects on the retina


Ensembles d'études connexes

Linear Algebra Exam 2 - Terms and Concepts

View Set

Health Assessment- PrepU Chapter 15 Assessing Head and Neck

View Set

Health Insurance Policy Provisions, Clauses, & Riders

View Set

Lab 7 Pre-Reading Joints Connect

View Set

Chapter 15: Implementation and Evaluation

View Set

difference between heliocentric and geocentric models

View Set

chapter 1 a and p review questions

View Set